Sei sulla pagina 1di 75

Master Your Physics - NEET / AIIMS KINEMATICS -1

PHYSICS
(For All Medical Entrance Exams)

KINEMATICS - 1
PMT SYLLABUS
Frame of reference. Motion in a straight line: Position -time graph, speed and velocity. Uniform
and non-unifor motion, average speed and instantaneous velocity. Uniformly accelerated motion,
velocity-time, position-time graphs, relations for uniformly accelerated motion (graphical treatment).
Elementary concepts of differentiation and integration for describing motion.

CONTENTS

1. Theory 3-46

2. Exercise 1 (Theoretical Questions) 47-51

3. Exercise 2 (Objective Questions) 52-63

5. Exercise 3 (Previsouly Asked) 64-69

4. Exercise 4 (Assertion & Reason) 70-72

4. Answers 73-75

Page # 1 Tarun Mankad (M.Tech. - IIT Kanpur)


Master Your Physics - NEET / AIIMS KINEMATICS -1

Page # 2 Tarun Mankad (M.Tech. - IIT Kanpur)


Master Your Physics - NEET / AIIMS KINEMATICS -1
KINEMATICS

1. INTRODUCTION
The branch of physics which deals with the study of motion of material objects is called mechanics.
Mechanics is divided into following branches.
(i) Statics :
Statics is the branch of mechanics which deals with the study of motion of objects under the effect of
forces in equilibrium.
(ii) Kinematics :
It is that branch of mechanics which deals with the study of motion of object without taking
into account the factors (i.e. nature of forces, nature of bodies etc.) which cause motion. Here
time factor plays an essential role.
(iii) Dynamics :
It is that branch of mechanics which deals with the study of motion of objects taking into account the
factors which cause motion.
Presently we shall study kinematics.
Rest : An object is said to be at rest if it does not change its position with time, with respect to its
surroudings. A book lying on a table, a person sitting in a chair are the examples of rest.
Motion : An object is said to be in motion if it changes its position with time, with respect
to its surroundings.
Example : A bird flying in air, a train moving on rails, a ship sailing on water, a man walking on
road are some of the examples of motion, visible to the eye. Motion of gas molecules is an example
of motion, invisible to the eye.
Rest & Motion are relative terms :
When we say that an object is at rest or in motion,then this statement is incomplete and meaningless.
Basically, rest & motion are relative terms. An object which is at rest can also be in motion simul-
taneously. This can be illustrated as follows.
The passengers sitting in a moving bus are at rest with respect to each other but they are also in
motion at the same time with respect to the objects like trees, buildings on the road side. So the
motion and rest are relative terms.
Rectilinear motion :
If a particle moves in a fixed direction, the motion of this type is called rectilinear motion or one
dimensional motion.
For example the motion of an ant on a wire is a rectilinear motion.
Two dimensional motion :
If the motion of a particle is in such a way that its position remains on a fixed plane, then the motion of
a particle is called two dimensional motion.
For example the motion of a rolling ball on a horizontal plane or earth’s surface, is a two
dimensional motion.
Three dimensional motion :

Page # 3 Tarun Mankad (M.Tech. - IIT Kanpur)


Master Your Physics - NEET / AIIMS KINEMATICS -1
If a line or plane is not fixed for the motion of a particle, then its motion is called three dimensional
motion.
For example the motion of a flying bird is a three dimensional motion.
2. DISTANCE
The length of the actual path between initial & final positions of a particle in a given interval of time is
called distance covered by the particle. Distance is the actual length of the path. It is the characteristic
property of any path ie., path is always associated when we consider distance between two positions.
Distance beteen A & B while moving through path (1) may or may not be equal to the distance between
A & B while moving through path (2)

II
Characteristics of Distance :
(i) It is a scalar quantity.
(ii) It depends on the path. A I B

(iii) It never reduces with time.


(iv) Distance covered by a particle is always positive & can never be negative or zero.
0 1 0
(v) Dimension :  M L T 

(vi) Unit : In C.G.S.system centimeter (cm), In S.I. system metre (m).


3. DISPLACEMENT
The shortest distance from the initial position to the final position of the particle is called displacement.
The displacement of a particle is measured as the change in the position of the particle in a particular
direction over a given time interval. It depends only on final & initial positions.
Displacement of a particle is a position vector of its final position w.r.t. initial position.

Position vector of A w.r.t. O  OA
A
 Y
 rA  x1 î  y1 ĵ  z1 k̂
B


Position vector of B w.r.t. O  OB
 O X
 rB  x 2 ˆi  y 2 ˆj  z 2kˆ

Displacement  AB   x 2  x1  ˆi   y 2  y1  ˆj   z 2  z1  kˆ Z
Characteristics of Displacement :
(i) It is a vector quantity.
(ii) The displacement of a particle between any two points is equal to the shortest distance between
them.
(iii) The displacement of an object in a given time interval can be positive, negative or zero.
0 1 0
(iv) Dimension : M L T 

(v) Unit : In C.G.S. centimeter (cm), In S.I. system meter (m).

Page # 4 Tarun Mankad (M.Tech. - IIT Kanpur)


Master Your Physics - NEET / AIIMS KINEMATICS -1

4. COMPARATIVE STUDY OF DISPLACEMENT AND DISTANCE

S.No. Displacement Distance

1. It has single value bewteen two points It may have more than one value between two points

2. 0  Displacement  0 Distance > 0

3. Displacement can decrease with time It can never decrease with time.

4. It is a vector quantity It is a scalar quantity

Special note:
1. The actual distance travelled by a particle in the given interval of time is always equal to or greater
than the magnitude of the displacement and in no case, it is less than the magnitude of the displacement,
i.e., Distance  | Displacement |
2. Displacement may be + ve, – ve or zero.
3. Distance, speed and time can never be negative.
4. At the same time particle cannot have two positions.
Some Impossible graphs :
Displacement

+
Distance

O
speed

– time time

O t1 time

Example 1.
An old person moves on a semi circular track of radius 40 m during a morning walk. If he starts
at one end of the track and reaches at the other end. Find the displacement of the person.
Sol. Displacement = 2R = 2 x 40 = 80 meter.
Example 2.
An athelete is running on a circular track of radius 50 meter. Calculate the displacement of the athlete
after completing 5 rounds of the track.
Sol. Since final and initial positions are same .
Hence displacement of athlete will be r  r  r  0
Example 3.
If a particle moves from point A to B then distance covered by particle will be.
Sol. D = x + 2x = 3x
Example 4.

A monkey is moving on circular path of radius 80 m. Calcualte the distance covered by the

Page # 5 Tarun Mankad (M.Tech. - IIT Kanpur)


Master Your Physics - NEET / AIIMS KINEMATICS -1
monkey.
Sol. Distance = Circumference of the circle
D  2R  D  2  80  160  3.14  502 .40m
Example 5.
A man has to go 50 m due north, 40 m due east and 20 m due south to reach a field.
(a) What distance he has to walk to reach the field?
(b) What is his displacement from his house to the field? N
Sol. Let origin be O then
W E
(a) Distance covered = OA + AB + BC = 50 + 40 + 20
= 110 meter S

(b) First method : Second method :



Displacement OC = OD2  CO 2 Displacement d  50ˆj  40iˆ  20ˆj
40m B
A
= 402  30 2 = 30 ˆj  40 ˆi
20m
 50m C
= 10 25  50 meter | d |  40 2  30 2 = 50 meter

O D

Example 6.
1
A body covers th part of a circular path. Calulate the ratio of distance and displacement.
4
Sol. Distance = AB from path (1) Displacement = AB
B
2r r 1
=  = OA 2  OB2
4 2
A
= O r
r2  r2  r 2

Distance r / 2 
  
Displacement r 2 2 2
Example 7.
A point P consider at contact point of a wheel on ground which rolls on ground without sliping then value
of displacement of point P when wheel completes half of rotation - [If radius of wheel is 1 m]
Sol. Displacement =  2  4 Ans.
P

2r
P
r

Page # 6 Tarun Mankad (M.Tech. - IIT Kanpur)


Master Your Physics - NEET / AIIMS KINEMATICS -1

Practice Problem - 1
Theoretical Questions
(A) 10 2m (B) 10m
1. Can displacement be greater than distance trav-
elled by an object ? 10
(C) m (D) 10  2m
2
2. Under what condition will the distance and dis-
placement of a moving object will have the 7. A man goes 10m towards North, then 20m
same magnitude ? towards east then displacement is
3. Can a body have a velocity and speed equal ? (A) 22.5m (B) 25m

4. Is magnitude of the displacement of an object (C) 25.5m (D) 30m


and total distance covered by it in certain time 8. A person moves 30 m north and then 20 m
interval same ? Explain. towards east and finally 30 2 m in south-
5. Here are three pairs of initial and final posi- west direction. The displacement of the person
tions, respectively, along an x axis. Which pairs from the origin will be
give a negative displacement : (A) 10 m along north (B) 10 m long south
(A) –3 m, +5 m; (B) –3 m, –7 m; (C) 7m, –3 m (C) 10 m along west (D) Zero
? 9. An aeroplane flies 400 m north and 300 m
Objective Questions south and then flies 1200 m upwards then net
displacement is
6. A Body moves 6 m north. 8 m east and 10m
vertically upwards, what is its resultant (A) 1200 m (B) 1300 m
displacement from initial position (C) 1400 m (D) 1500 m
*****

5. SPEED
Speed of an object is defined as the time rate of change of position of the object in any direction.
It is measured by the distance travelled by the object in unit time in any direction. i.e.,

distance travelled
speed=
time taken
(i) It is a scalar quantity.
(ii) It gives no idea about the direction of motion of the object.
(iii) It can be zero or positive but never negative.
(iv) Unit: C.G.S. cm/sec, S.I. m/sec,

100 5 5
1km / h =  m/s 1 km / h  m/s
60  60 18 18

(v) Dimension : M0 L1 T 1
Types of speed :

Page # 7 Tarun Mankad (M.Tech. - IIT Kanpur)


Master Your Physics - NEET / AIIMS KINEMATICS -1
(a) Uniform speed : An object is said to be moving with a uniform speed, if it covers equal distances
in equal intervals of time, how soever small these intervals may be. The uiform speed is shown by
straight line in distance time graph.
For example: suppose a train travels 1000 metre in 60 second. The train is said to be moving with
uniform speed, if it travels 500 metre in 30 second, 250 metre in 15 second, 125 metre in 7.5 second
and so on.
(b) Non Uniform Speed : An object is said to be moving with a variable speed if it covers equal distance
in unequal intervals of time or unequal distances in equal intervals of time, howsoever small these
intervals may be.
For example : suppose a train travels first 1000 metre in 60 second, next 1000 metre in 120 second
and next 1000 metre in 50 second, then the train is moving with variable speed.
(c) Average Speed : When an object is moving with a variable speed, then the average speed of the
object is that constant speed with which the object covers the same distance in a given time as it
does while moving with variable speed during the given time. Average speed for the given motion
is defined as the ratio of the total distance travelled by the object to the total time take i.e.,

total distance travelled


Average speed V =
total time taken
Note : If any car covers distance x1, x2, ........... in the time intervals t1, t2, ....... then.

x1  x 2  x 3  .......  x n
V=
t1  t 2  .....  t n

SOME IMPORTANT CASES RELATED TO AVERAGE SPEED :


Case : 1
If body covers distances x1, x2, and x3 with speeds v1, v2, and v3 respectively in same direction then
average speed of body.

x1  x 2  x 3 x x2 x
 V here, t1  1 , t 2  , t3  3
t1  t 2  t 3 v1 v2 v3

x1  x 2  x 3 V1 V2 V3
V
x1 x 2 x 3 x
  x'
v1 v 2 v 3 O x1 x2 x3

If body covers equal distances with different speeds then, x1  x 2  x 3  x

3x 3 3v1v 2 v 3
V  
x1 x x 1 1 1 v1v 2  v 2 v 3  v 3 v1
   
v1 v 2 v 3 v1 v 2 v 3

Case : 2
If any body travels with speeds v1,v2,v3 during time intervals t1,t2,t3 respectively then the average speed
of the body wil be

x1  x 2  x 3 v1t1  v 2 t 2  v 3 t 3
Average speed V  t  t  t  t1  t 2  t 3 If t1  t 2  t 3  t
1 2 3

Page # 8 Tarun Mankad (M.Tech. - IIT Kanpur)


Master Your Physics - NEET / AIIMS KINEMATICS -1


 v1  v 2  v 3   t   v1  v 2  v 3 
3t 3
(d) Instantaneous speed :
The speed of the body at any instant of time or at a particular position is called instantaneous speed.

x
Let a body travel a distance  x in the time interval  t , then its average speed  .
t
When t  0 , then average speed of the body becomes the instantaneous speed.

x dx
Instantaneous speed =Lim  (to be studied later)
t 0 t dt
Note :
(a) Speedometer of the vehicle measures its instantaneous speed.
(b) In case of a uniform motion of an object, the instantaneous speed is equal to its uniform speed.
6. VELOCITY
It is defined as rate of change of displacement.
Characteristics of Velocity :
(i) It is a vector quantity. (ii) Its direction is same as that of displacement.
(iii) Unit and dimension : Same as that of speed.
Types of Velcoity :
(a) Instantaneous Velocity (b) Average Velocity
(c) Uniform Velocity (d) Non-uniform Velocity
(a) Instantaneous Velocity : It is defined as the velocity at some particular instant.
 
 r dr
Instantaneous velocity  lim
t 0 t
 (to be studied later)
dt

Total Displacement
(b) Average Velocity : Average Velocity =
Total time
(c) Uniform Velocity : A particle is said to have uniform velocity, if magnitudes as well as direction
of its velocity remains same and this is possible only when the particles moves in same straight line
reservering its direction.
(d) Non-uniform Velocity : A particle is said to have non-uniform velocity, if either of magnitude or
direction of velocity changes (or both changes).
7. COMPARATIVE STUDY OF SPEED AND VELOCITY

S.No. Speed Velocity


1. It is the time rate of change of distance of a body. It is the time rate of charge of displacement
of a particle
2. It tells nothing about the direction of motion It tells the direction of motion of the particle
of the particle
3. It can be positive or zero It can be positive or negative or zero
4. It is a scalar quantity It is a vector quantity

Page # 9 Tarun Mankad (M.Tech. - IIT Kanpur)


Master Your Physics - NEET / AIIMS KINEMATICS -1

Example 8.
A car travels a distance A to B at a speed of 40 km/h and returns to A at a speed of 30 km/h.
(i) What is the average speed for the whole journey?
(ii) What is the average velocity ?

s
Sol. (i) Let AB = s, time taken to go from A to B, t1 = h
40

s
and time taken to go from B to A, t2 = h
30

s

s

 3  4  s  7s h
 total time taken = t1 + t2 =
40 30 120 120

Total distance travelled = s + s = 2s

total distance travelled 2s 120  2


 Average speed     34.3 km / h
total time taken 7s /120 7

(ii) Total displacement = zero, since the car returns to the original position.

total displacement 0
Therefore, average velocity   0
time taken 2t

Example 9.
From the adjoining position time graph for two particles A and B the ratio of velocities vA : vB will
be

[1] 1 : 2 [2] 1: 3 [3] 3 :1 [4] 1 : 3

VA tan a tan 300 1/ 3 1


Sol.     Ans. [4]
VB tan b tan 600 3 3

Example 10.
In a car race, car A takes a time of t sec. less than car B at the finish and passes the finishing point
with a velocity  m/s more than the car B. Assuming that the cars start from rest and travel with

constant accelerations a1 and a2 respectively, show that v=  a1a2  t

Sol. Let the time taken by two cars to complete the journey be t1 and t2 and their velocities at the finish
be 1 and 2 respectively..

Given that, t1  t 2  t and 1  2   ............ (1)

11 2 11
Now S1  sS1  a1t1 and Ss22 Ss  a2 t 22 ............ (2)
22 22

(At start, v1 = v2 = 0) ............ (3)

Page # 10 Tarun Mankad (M.Tech. - IIT Kanpur)


Master Your Physics - NEET / AIIMS KINEMATICS -1
Hence a1t12  a2 t 22  2s B

Displacement
A

Also, v 1  a1t1 and v 2  a 2 t 2 60º

or v1t1 = a1t12 = 2s and v2t2 = a2t22 = 2s 30º


O Time
2s 2s
t1  t2 
 v 1 and v2

 1 1
So, t 2  t1  2s    ............ (4)
 v 2 v1 

From equations (1) and (4) we have

 1 1
2s   t
 v 2 v1 

 v1  v 2   v 
or 2s    t or 2s  t
 v1v 2   v 1v 2 

 v 1v 2   v 2 v 2 
or v    t   1 22   t
 2s   (2s) 

 v1v 2 
  × t = (a a )  t
 t1t 2  1 2

Page # 11 Tarun Mankad (M.Tech. - IIT Kanpur)


Master Your Physics - NEET / AIIMS KINEMATICS -1

Practice Problem - 2
Theoretical Questions 8. An automobile travels on a straight road for
1. Can an object have constant speed but variable 40 km at 30 km/h. It then continues in the same
velocity ? direction for another 40km at 60 km/h.
2. Can the speed of a body be negative ? (A) What is the average velocity of the car
3. Can a body have a velocity and speed equal ? during this 80 km trip ? (Assume that it moves
in the positive x direction.)
4. Show that average velocity of the object over
an interval of time is either smaller than or (B) What is the average speed ?
equal to the average speed of the object over 9. Compute your average velocity in the following
the same interval. two cases :
5. An athlete completes one round of a circular (A) You walk 73.2 m at a speed of 1.22 m/s
track of radius R in 40 seconds. What will be and then run 73.2 m at a speed of 3.05 m/s
the displacement at the end of 2 min. 20 along a straight track.
second ? (B) You walk for 1.00 min at a speed of 1.22
6. You drive a beat-up pickup truck along a m/s and then run for 1.00 min at 3.05 m/s along
straight road for 8.4 km at 70 km/h, at which a straight track.
poin the truck runs out of gasoline and stops. Objective Questions
Over the next 30 min, you walk another 2.0
10. A car travels from A to B at a speed of
km farther along the road to a gasoline station.
20 km / hr and returns at a speed of 30 km / hr .
(A) What is your overall displacement from the The average speed of the car for the whole
beginning of your drive to your arrival at the journey is
station ? (A) 25 km / hr (B) 24 km / hr
(B) What is the time interval t from the beginning (C) 50 km / hr (D) 5 km / hr
of your drive to your arrival at the station ?
11. A boy walks to his school at a distance of 6
km with constant speed of 2.5 km/hour and
(C) What is your average velocity avg from the walks back with a constant speed of 4 km/hr.
beginning of your drive to your arrival at the His average speed for round trip expressed in
station? Find it both numerically and km/hour, is
graphically. (A) 24/13 (B) 40/13
(C) 3 (D) 1/2
7. Figure shows four paths along which objects
12. A car travels the first half of a distance between
move from a starting point to a final point, all
two places at a speed of 30 km/hr and the
in the same time. The paths pass over a grid of second half of the distance at 50 km/hr. The
equally spaced straight lines. Rank the paths average speed of the car for the whole journey
according to is
(A) 42.5 km/hr (B) 40.0 km/hr
(A) the average velocity of the objects and (B)
the average speed of the objects, greatest first. (C) 37.5 km/hr (D) 35.0 km/hr
13. One car moving on a straight road covers one
third of the distance with 20 km/hr and the
rest with 60 km/hr. The average speed is
(A) 40 km/hr (B) 80 km/hr
2
(C) 46 3 km/hr (D) 36 km/hr

Page # 12 Tarun Mankad (M.Tech. - IIT Kanpur)


Master Your Physics - NEET / AIIMS KINEMATICS -1
14. A car moves for half of its time at 80 km/h th
18. If a car covers 2/5 of the total distance with
and for rest half of time at 40 km/h. Total v1 speed and 3/5th distance with v2 then average
distance covered is 60 km. What is the average speed is
speed of the car
(A) 60 km / h (B) 80 1 v1  v 2
(A) 2 v1v 2 (B) 2
(C) 120 (D) 180
15. A train has a speed of 60 km/h. for the first
one hour and 40 km/h for the next half hour. 2v1v 2 5v1v 2
Its average speed in km/h is (C) v  v (D) 3v  2v
1 2 1 2

(A) 50 (B) 53.33


19. The numerical ratio of displacement to the
(C) 48 (D) 70 distance covered is always
16. A 150 m long train is moving with a uniform
velocity of 45 km/h. The time taken by the train (A) Less than one
to cross a bridge of length 850 meters is
(B) Equal to one
(A) 56 sec (B) 68 sec
(C) 80 sec (D) 92 sec (C) Equal to or less than one
17. A particle is constrained to move on a straight (D) Equal to or greater than one
line path. It returns to the starting point after
10 sec. The total distance covered by the 20. A particle moves for 20 seconds with velocity
particle during this time is 30 m. Which of the 3 m/s and then velocity 4 m/s for another 20
seconds and finally moves with velocity 5 m/
following statements about the motion of the
s for next 20 seconds. What is the average
particle is false velocity of the particle
(A) Displacement of the particle is zero
(A) 3 m/s (B) 4 m/s
(B) Average speed of the particle is 3 m/s
(C) Displacement of the particle is 30 m (C) 5 m/s (D) Zero
*******
(D) Both (A) and (B)

8. ACCELERATION
It is defined as the rate of change of velocity.
(i) It is a vector quantity.
(ii) Its direction is same as that of change in velocity and not of the velcoity (That is why, acceleration
in circular motion is towards the centre).
(iii) There are three ways possible in which change in velocity may occur.

W hen both the direction


W hen only m agnitude
W hen only direction changes and m agnitude
changes
change
To change the direction, In this case, net force or net In this case, net force or
net acceleration or net acceleration should be net acceleration has two
force should be parallel or antiparallel to the com ponents. O ne
perpendicular to direction direction of velocity. com ponent is parallel or
of velocity. (straight line m otion) antiparallel to velocity and
another one is
perpendicular to velocity.

Ex: Uniform circular Ex: W hen ball is thrown up Ex: Projectile m otion
m otion. under gravity.

Page # 13 Tarun Mankad (M.Tech. - IIT Kanpur)


Master Your Physics - NEET / AIIMS KINEMATICS -1
Types of acceleration -
(a) Instantaneous acceleration : It is defined as the acceleration of a body at some particular instant.

 
v dv
Instantaneous acceleration = lim
t 0 
t dt

  
  v v 2  v1
(b) Average acceleration = a av  
t t 2  t1

(c) Uniform acceleration : A body is said to have uniform acceleration if magnitude and direction
of the acceleration remains constant during particle motion.
Note : If a particle is moving with uniform acceleration, this does not necessarily imply that particle
is moving in straight line.
Example : Parabolic motion (to be studied later)
(d) Non-uniform acceleration : A body is said to have non-uniform acceleration, if magnitude or
direction or both, change during motion.
Note :
(i) Acceleration is a vector with dimensions [LT–2] and SI units (m/s2)
(ii) If acceleration is zero, velocity will be constant and motion will be uniform.
(iii) However if acceleration is constant acceleration is uniform but motion is non-uniform and if
acceleration is not constt. both motion and acceleration are non-uniform.
  
(iv) If a force F acts on a particle of mass m then by Newton’s law a  F / m

   
ds  dv d  ds  d2 s
(v) As by definition , v  so, a    (to be studied later)
dt dt dt  dt  dt 2
 


i.e. if s is given as a function of time, second time derivative of displacement gives acceleration.

dv dv dx dv
(vi) If velocity is given as function of position then by the chain rule a   .  av
dt dx dt dx

dx
[as =v]
dt

 
 dv  dv
(vii) As acceleration a  , the slope of velocity time graph gives acceleration i.e. a  = tan
dt dt

(viii) Acceleration can be positive or negative. Positive acceleration means velocity is increasing with
time while negative acceleration called retardation means velocity is decreasing with time.

Page # 14 Tarun Mankad (M.Tech. - IIT Kanpur)


Master Your Physics - NEET / AIIMS KINEMATICS -1

Practice Problem - 3
Theoretical Questions Objective Questions
1. The direction in which an object moves is given 11. The correct statement from the following is
by the direction of velocity of the object and
not by the direction of acceleration. Explain (A) A body having zero velocity will not
this statement with suitable example. necessarily have zero acceleration

2. Can a body have zero velocity and still be ac- (B) A body having zero velocity will
celerating ? necessarily have zero acceleration

3. Can the direction of velocity of a body change, (C) A body having uniform speed can have
when acceleration is constant ? only uniform acceleration

4. A car manufacturer advertises that the brakes (D) A body having non-uniform velocity will
are so perfect that the car stops instantaneously. have zero acceleration
Comment. 12. For a moving body at any instant of time
5. Which of the two; velocity or acceleration de- (A) If the body is not moving, the acceleration
cides the direction of motion of a body ? Ex- is necessarily zero
plain with the help of an example.
(B) If the body is slowing, the retardation is
6. Give examples to show that a positive accel- negative
eration can be associated with a ‘slowing down’
and a negative acceleration with a ‘speeding (C) If the body is slowing, the distance is
up’. negative

7. Can an object be accelerated without speeding (D) If displacement, velocity and acceleration
up or slowing down ? at that instant are known, we can find the
displacement at any given time in future
Subjective Questions
13. Acceleration of a particle changes when
8. A wombat moves along an x axis. What is the
sign of its acceleration if it is moving (a) in the (A) Direction of velocity changes
positive direction with increasing speed, (b) in (B) Magnitude of velocity changes
the positive direction with decreasing speed,
(c) in the negative direction with increasing (C) Both of above
speed, and (d) in the negative direction with (D) Speed changes
decreasing speed.
******
9. A particle had a speed of 18 m/s at a certain
time, and 2.4 s later its speed was 30 m/s in
the opposite direction. What were the magni-
tude and direction of the average acceleration
of the particle during this 2.4 s interval ?

10. An automobile driver increases the speed at a


constant rate from 25 km/h to 55 km/h in 0.50
min. A bicycle rider speeds up at a constant
rate from rest to 30 km/h in 0.50 min. Calcu-
late their accelerations ?

Page # 15 Tarun Mankad (M.Tech. - IIT Kanpur)


Master Your Physics - NEET / AIIMS KINEMATICS -1
9. Interpretation of Motion Using Graphs
During motion of the particle its parameters of kinematical analysis (v, a, s) changes with time. This
can be represented on the graph.
Position Time Graph
Position time graph is plotted by taking time t along x-axis and position of the particle on y-axis.

Let AB is a position-time graph for any moving particle


Change in position y 2  y1
As Velocity =  …(i) y
Time taken t 2  t1

Velocity
v2 D B
BC AD y 2  y1
From triangle ABC, tan     ….(ii)
AC AC t 2  t1 v1 
C
A
By comparing (i) and (ii) Velocity = tan
x
v = tan O t1 t2
Time

It is clear that slope of tangent on position-time graph represents the velocity of the particle.

1.  = 0° so v = 0 i.e., line parallel to time axis represents that the particle is


O T

at rest.

  = 90° so v =  i.e., line perpendicular to time axis represents that particle is


O T

changing its position but time does not changes it means the particle possesses infinite velocity. Practically
this is not possible.

3.  = constant so v = constant, a = 0 i.e., line with constant slope represents uniform


O T

velocity of the particle.

  is increasing so v is increasing, a is positive.i.e., line bending towards position


O T

axis represents increasing velocity of particle. It means the particle possesses acceleration.

Page # 16 Tarun Mankad (M.Tech. - IIT Kanpur)


Master Your Physics - NEET / AIIMS KINEMATICS -1

5.  is decreasing so v is decreasing, a is negative i.e., line bending towards time


O T

axis represents decreasing velocity of the particle. It means the particle possesses retardation.

   constant but > 90o so v will be constant but negative i.e., line with negative
O T

slope represent that particle returns towards the point of reference. (negative displacement).

P
A B
C
7. Straight line segments of different slopes represent that velocity of the body
O T
S

changes after certain interval of time.

T
8. O This graph shows that at one instant the particle has two positions, which is

not possible.

9. T The graph shows that particle coming towards origin initially and after that it is
O

moving away from origin.

Note : If the graph is plotted between distance and time then it is always an increasing curve and it
never comes back towards origin because distance never decrease with time. Hence such type of distance
time graph is valid up to point A only, after point A, it is not valid as shown in the figure.

A
Distance

O Time
Fig. 2.9

Page # 17 Tarun Mankad (M.Tech. - IIT Kanpur)


Master Your Physics - NEET / AIIMS KINEMATICS -1
Velocity-time Graph
The graph is plotted by taking time t along x-axis and velocity of the particle on y-axis.
Calculation of Distance and displacement : The area covered between the velocity time graph and
time axis gives the displacement and distance travelled by the body for a given time interval.
Total distance | A1 || A2 || A3 |

= Addition of modulus of different area. i.e. s  | |dt

Total displacement  A1  A2  A3
= Addition of different area considering their sign.
i.e. r    dt
Area above time axis is taken as positive, while area below time axis is taken as negative

+

1 3
t
2

–

Table : Various velocity -time graphs and their interpretation


Velocity

  = 0°, a = 0, v = constanti.e., line parallel to time axis represents that the particle is
O

moving with constant velocity.


Velocity

2.  = 90o, a = , v = increasing i.e., line perpendicular to time axis represents that the
O
Time

particle is increasing its velocity, but time does not change. It means the particle possesses infinite
acceleration. Practically it is not possible.
Velocity

3.  = constant, so a = constant and v is increasing uniformly with time i.e., line with
O Time

constant slope represents uniform acceleration of the particle.

Page # 18 Tarun Mankad (M.Tech. - IIT Kanpur)


Master Your Physics - NEET / AIIMS KINEMATICS -1

Velocity
4.  increasing so acceleration increasing i.e., line bending towards velocity axis represent
O Time

the increasing acceleration in the body.


Velocity

5.  decreasing so acceleration decreasing i.e. line bending towards time axis represents
O Time

the decreasing acceleration in the body


Velocity

6. Positive constant acceleration because  is constant and < 90o but initial velocity
O
Time

of the particle is negative.


Velocity

7. Positive constant acceleration because  is constant and < 90o but initial velocity of
O
Time

particle is positive.
Velocity

9. Negative constant acceleration because  is constant and > 90o but initial velocity of
O
Time

the particle is positive.


Velocity

10. O
Negative constant acceleration because  is constant and > 90o but initial velocity
Time

of the particle is zero.


Velocity

11. O
Time Negative constant acceleration because  is constant and > 90o but initial velocity of

the particle is negative.

Page # 19 Tarun Mankad (M.Tech. - IIT Kanpur)


Master Your Physics - NEET / AIIMS KINEMATICS -1
Example 11.
Describe the motion shown by the following velocity-time graphs.

(a) (b)

Sol.(a) During interval AB: velocity is +ve so the particle is moving in +ve direction, but it is slowing down
as acceleration (slope of v-t curve) is negative. During interval BC: particle remains at rest as velocity
is zero. Acceleration is also zero. During interval CD: velocity is -ve so the particle is moving in -ve
direction and is speeding up as acceleration is also negative.
(b) During interval AB: particle is moving in +ve direction with constant velocity and acceleration is zero.
During interval BC: particle is moving in +ve direction as velocity is +ve, but it slows down until it
comes to rest as acceleration is negative. During interval CD: velocity is -ve so the particle is moving
in -ve direction and is speeding up as acceleration is also negative.
Important Points to Remember

 For uniformly accelerated motion (a  0), xt graph is a parabola (opening upwards if a > 0 and
opening downwards if a < 0). The slope of tangent at any point of the parabola gives the velocity at that
instant.
 For uniformly accelerated motion (a  0), vt graph is a straight line whose slope gives the acceleration
of the particle.
 In general, the slope of tangent in xt graph is velocity and the slope of tangent in vt graph is the
acceleration.
 The area under at graph gives the change in velocity.
 The area between the vt graph gives the distance travelled by the particle, if we take all areas as
positive.
 Area under vt graph gives displacement, if areas below the taxis are taken negative.

Example 12.
For a particle moving along x-axis, velocity-time graph is as shown in figure. Find the distance travelled
and displacement of the particle?
Sol. Distance travelled = Area under v-t graph (taking all areas as +ve.)
 Distance travelled = Area of trapezium + Area of triangle
1
= 2  6 8 + 1  4  5
2 2
= 32 + 10 = 42 m
Displacement = Area under v-t graph (taking areas below time axis as -ive.)
 Displacement = Area of trapezium  Area of triangle
1
= 2  6 8  1  4  5
2 2
= 32  10 = 22 m
Hence, distance travelled = 42 m and displacement = 22 m.

Page # 20 Tarun Mankad (M.Tech. - IIT Kanpur)


Master Your Physics - NEET / AIIMS KINEMATICS -1

Practice Problem - 4
Subjective Questions 10. Find the angle between the x-axis and the line
1. Find the inclination of the line whose slope joining the points (,3 –1) and (4, –2).
11. Consider the following population and year
1
is  . graph (Fig.), find the slope of the line AB and
3
using it, find what will be the population in the
2. Find the slope of the line through the points year 2010 ?
(4, –6), (–2, –5).
3. Determine  , so that 2 is the slope of the

line through (2, 5) and  ,3 .

4. Show that the line joining the points (2, –3)


and (–5, 1) is parallel to the line joining (7,
–1) and (0, 3).
5. Find the equation of the straight line paral- 12. Write the equations for the x and y-axes.
lel to y-axis and at a distance 13. Passing through the point (–4, 3) with slope

(i) 3 units to the right (ii) 2 units to the 1


.
left. 2

6. Find the equation of the straight line paral- 14. Passing through (0, 0) with slope m.

lel to x-axis and at a distance 15. Find angles between the lines 3 x  y 1 and
(i) 5 units above the x-axis (ii) 9
x  3 y 1.
units below the x-axis.
Objective Questions
7. Find the equation of the straight line which
passes through the point (2, –3) and is 16.The acceleration of a moving body can be found
(i) parallel to the x-axis from
(A) Area under velocity-time graph
(ii) perpendicular to the x-axis
(B) Area under distance-time graph
8. Find the slope of the lines :
(C) Slope of the velocity-time graph
(a) Passing through the points (3, –2) and (–1, 4),
(D) Slope of distance-time graph
(b) Passing through the points (3, –2) and (7, –2), 17. A man goes 10m towards North, then 20m
(c) Passing through the points (3, –2), and (3, 4), towards east then displacement is
o
(d) Making inclination of 60 with the positive di- (A) 22.5m (B) 25m
rection of x-axis. (C) 25.5m (D) 30m
9. Find the slope of the line, which makes an angle
of 30o with the positive direction of y-axis mea-
sured anticlockwise.

Page # 21 Tarun Mankad (M.Tech. - IIT Kanpur)


Master Your Physics - NEET / AIIMS KINEMATICS -1

Practice Problem - 5
Theoretical Questions angles of 30° and 45° with the time axis. What
1. An object is in uniform motion along a straight is ratio of the velocities vA : vB ?
line. What will be position-time graph for the 11. How is the velocity-time graph of accelerated
motion of the object if (a) x0 = +ve, v = +ve motion helpful in studying the motion of the
(b) x0 = +ve, v = –ve (c) x0= –ve, v = +ve and object in one dimension?
(d) both x0 and v are negative? The letters x0
12. How is the position-time graph of uniformly
and v represent position of the object at time t
accelerated motion in one dimension helpful
= 0 and uniform velocity of the object respec-
in studying the motion of the object?
tively.
13. What is the acceleration of a body when its
2. Is the time variation of position, shown in fig-
velocity-time graph is (i) perpendicular to time
ure observed in nature?
axis. (ii) parallel to time axis?
TIME 14. What do the slopes of ‘distance - time’ and
‘velocity-time’ graph represent? What do posi-
tive and negative values of these slopes im-
ply?
POSITION
15. How can one determine (i) the distance (ii) the
displacement covered by a uniformly acceler-
3. Two straight lines drawn on the same displace- ated body from its velocity-time graph?
ment-time graph make angles 30º and and 60° 16. Figure shows the x coordinate of a particle as
with time-axis respectively fig. Which line rep- a function of time. Find the signs of vx and ax
resents greater velocity? What is the ratio of at t=t1, t=t2 and t=t3.
two velocities?
x

DISPLACEMENT
B

60° A

O
30°
TIME
17. The velocity-time graph of a body moving in a
4. What will be nature of velocity-time graph for straight line is shown in fig. Find the displace-
a uniform motion? ment and the distance travelled by the body in
5. What will be the nature of position-time graph 6 seconds.
for a uniform motion?
6. What does slope of position-time graph repre-
sent for a uniform motion?
7. If the displacement-time graph of a particle is
parallel to (a) displacement axis (b) the time
axis, what will be the velocity of the particle?
8. Can position-time graph have negative slope?
18. The velocity-time graph of a particle moving
9. Draw position - time graph for a stationary along a straight line is as shown in fig. Calcu-
object. late the distance covered between t=0 to t=10
10. The displacement-time graph for the two par- seconds. Also find displacement in time 0 to
ticles A and B are straight lines inclined at 10 seconds.

Page # 22 Tarun Mankad (M.Tech. - IIT Kanpur)


Master Your Physics - NEET / AIIMS KINEMATICS -1
v ( m/s) OA, AB, BC, CD
20
(A) + 0 + + (B) – 0 + 0
10
(C) + 0 – +(D) – 0 – 0
0 t(s)
2 4 6 8 10 22. The displacement-time graph of moving
-10
particle is shown below
-20
s
19. The velocity-time graph of a particle moving

Displacement
along a straight line is shown in the fig. by D
F
curve OABCD. Calculate the distance covered C E
by the particle between (i) t=zero to t=18 sec-
onds (ii) t = 2s to t = 12s. and the maximum Time t

value of acceleration during this interval. The instantaneous velocity of the particle is
v (m/s)
negative at the point

20
A (A) D (B) F

15
(C) C (D) E

10
B C 23. Which of the following velocity-time graphs
shows a realistic situation for a body in motion
5

D v v
t(s)
O
2 4 6 8 10 12 14 16 18

Objective Questions (A) (B)


20. Mark the correct statements for a particle t t
going on a straight line:
(A) If the velocity and acceleration have v v
opposite sign, the object is slowing down.
(B) If the position and velocity have opposite
sign, the particle is moving towards the origin. (C) (D)
(C) If the velocity is zero at an instant, the t t

acceleration should also be zero at that instant.


24. Figure shows the displacement-time graph of
(D) If the velocity is zero for a time interval,
a particle moving on the X-axis.
the acceleration is zero at any instant within
the time interval.
21. The graph between the displacement x and
time t for a particle moving in a straight line
is shown in figure. During the interval
OA, AB, BC and CD , the acceleration of the
particle is

Y (A) the particle is continuously going in posi-


tive x direction
Displacement

(B) the particle is at rest


D

C
(C) the velocity increases up to a time t0, and
A B
O X then becomes constant
Time t
(D) the particle moves at a constant velocity
up to a time t0, and then stops.
Page # 23 Tarun Mankad (M.Tech. - IIT Kanpur)
Master Your Physics - NEET / AIIMS KINEMATICS -1
25. The velocity-time plot for a particle moving
on a straight line is shown in the figure.

29. Figure shows the graph of the x-coordinate of


(A) The particle has a constant acceleration. a particle going along the X-axis as a function
(B) The particle has never turned around. of time. Find (a) the average velocity during 0
(C) The particle has zero displacement. to 10 s, (b) instantaneous velocity at 2, 5, 8
(D) The average speed in the interval 0 to 10 s and 12s.
is the same as the average speed in the interval
10 s to 20 s.
26. Figure shows the position of a particle moving
on the X-axis as a function of time.

30. From the velocity-time plot shown in figure,


find the distance travelled by the particle during
the first 40 seconds. Also find the average
velocity during this period.

(A) The particle has come to rest 6 times.


(B) The maximum speed is at t = 6 s.
(C) The velocity remains positive for t - 0 to t
= 6 s.
(D) The average velocity for the total period
shown is negative. 31. A particle starts from a point A and travels
along the solid curve shown in figure. Find
27. The speed of a car as a function of time is
approximately the position B of the particle
shown in figure. Find the distance travelled
such that the average velocity between the
by the car in 8 seconds and its acceleration.
positions A and B has the same direction as
the instantaneous velocity at B.

28. Figure shows the graph of velocity versus time


for a particle going along the X-axis. Find (a)
the acceleration, (b) the distance travelled in 0 32. The variation of velocity of a particle with time
to 10s and (c) the displacement in 0 to 10 s. moving along a straight line is illustrated in
the following figure. The distance travelled

Page # 24 Tarun Mankad (M.Tech. - IIT Kanpur)


Master Your Physics - NEET / AIIMS KINEMATICS -1
by the particle in four seconds is
4
 (m/s)
3
30 2
Velocity (m/s) 20 1
0
10 10 20 30 40 50 60
0 t (sec)
1 2 3 4
Time in second (A) 60 m (B) 50 m
(A) 60 m (B) 55 (C) 30 m (D) 40 m
(C) 25 (D) 30 36. For the velocity-time graph shown in figure
33. The v  t graph of a moving object is given in below the distance covered by the body in last
figure. The maximum acceleration is two seconds of its motion is what fraction of
the total distance covered by it in all the seven
seconds
80
Velocity (cm/sec)

60 10

Velocity (m/sec)
40 8
6
20
4
0
10 20 30 40 50 60 70 80 2
Time (sec.)
1 2 3 4 5 6 7
Time (sec)
(A) 1cm / sec c 2 (B) 2cm / sec 2 ( C )
3 cm / sec 2 (D) 6 cm / sec 2 1 1
(A) 2 (B) 4
34. A lift is going up. The variation in the speed
of the lift is as given in the graph. What is the 1 2
(C) 3 (D) 3
height to which the lift takes the passengers
Velocity (m/sec)

3.6

2 Time (sec) 10 12

(A) 3.6 m (B) 28.8


(C) 36.0 m
(D) Cannot be calculated from the above graph
35. Velocity-time (v-t) graph for a moving object
is shown in the figure. Total displacement of
the object during the time interval when there
is non-zero acceleration and retardation is

Page # 25 Tarun Mankad (M.Tech. - IIT Kanpur)


Master Your Physics - NEET / AIIMS KINEMATICS -1

Practice Problem - 6
Objective Questions
1. An object is moving with a uniform
100
acceleration which is parallel to its

Speed in km/hours
80
instantaneous direction of motion. The D
60
displacement (s)  velocity (v) graph of this 40
object is 20 B C

A N M L E
0.25 0.75 1.00 1.5 2.00
s s
Time in hours

(A) (B) (A) 140 km h–2 (B) 160 km h–2


v v (C) 100 km h–2 (D) 120 km h–2
4. Acceleration-time graph of a body is shown.
s s
The corresponding velocity-time graph of the
same body is
(C) (C)
a
v v

2. The graph of displacement v/s time is

v v

t
(A) (B)
Its corresponding velocity-time graph will be t t

v v
v v

(A) (C) (C) (D)


t t
t t

5. The acceleration-time graph of a body is shown


below
v v
a

(C) (D)
t
t

t
3. A train moves from one station to another in 2
hours time. Its speed-time graph during this
motion is shown in the figure. The maximum The most probable velocity-time graph of the
acceleration during the journey is body is

Page # 26 Tarun Mankad (M.Tech. - IIT Kanpur)


Master Your Physics - NEET / AIIMS KINEMATICS -1


1
 (A) m/s (B) 3 m/s
3

(A) (B) 1
(C) 3 m/s (D)
3
t t

7. The   t plot of a moving object is shown in


the figure. The average velocity of the object
 
during the first 10 seconds is

(C) (D) 5

Velocity (ms )
-1
t t Time (sec)
0
5 10
6. From the following displacement-time graph
find out the velocity of a moving body –5

(A) 0 (B) 2.5 ms–1


Time (sec)

(C) 5 ms–1 (D) 2 ms–1

30
o ****
O
Displacement (meter)

10. UNIFORMLY ACCELERATED MOTION


If a particle is accelerated with constant acceleration in an interval of time, then the motion is termed as
uniformly accelerated motion in that interval of time.
For uniformly accelerated motion along a straight line (xaxis) during a time interval of t seconds, the
following important results can be used.

(a) v = u + at

(b) s = ut + 1/2 at2


s = vt  1/2 at2
xf = xi + ut + 1/2 at2
(c) v2 = u2 + 2as
(d) s = 1/2 (u + v) t
(e) sn = u + a/2 (2n  1)
u = initial velocity (at the beginning of interval)
a = acceleration
v = final velocity (at the end of interval)
s = displacement (xf  xi)
xf = final coordinate (position)
xi = initial coordinate (position)
sn = displacement during the nth sec

Page # 27 Tarun Mankad (M.Tech. - IIT Kanpur)


Master Your Physics - NEET / AIIMS KINEMATICS -1
Example 13. A particle moving rectilinearly with constant acceleration is having initial velocity of 10
m/s. After some time, its velocity becomes 30 m/s. Find out velocity of the particle at the mid point of
its path?
Sol. Let the total distance be 2x.
 distance upto midpoint = x
Let the velocity at the mid point be v
and acceleration be a.
From equations of motion
v2 = 102 + 2ax ____ (1)
2 2
30 = v + 2ax ____ (2)
(2) - (1) gives

v2 - 302 = 102 - v2  v2 = 500  v = 10 5 m/s


Example 14.
A police inspector in a jeep is chasing a pickpocket an a straight road. The jeep is going at its maximum
speed v (assumed uniform). The pickpocket rides on the motorcycle of a waiting friend when the jeep
is at a distance d away, and the motorcycle starts with a constant acceleration a. Show that the pick
pocket will be caught if v  2ad .
Sol. Suppose the pickpocket is caught at a time t after motorcycle starts. The distance travelled by the
motorcycle during this interval is

1
s  at2 ____ (1)
2
During this interval the jeep travels a distance
s  d  vt ____ (2)

1 2
By (1) and (2), at  d  vt
2

v  v 2  2ad
or, t
a
The pickpocket will be caught if t is real and positive.
This will be possible if v 2  2ad or,, v 2ad
Example 15.
The velocity acquired by a body moving with uniform acceleration is 20 m/s in first 2 sec and 40
m/s in first 4 sec. Calculate initial velocity.

u v1 v2
v 2  v1
Sol. a  t  t
2 1 t=0 t1 = 2 sec t2 = 4 sec

40  20 20
a   10m / s2
42 2
Now, v  u  at

v1  u  at1

Page # 28 Tarun Mankad (M.Tech. - IIT Kanpur)


Master Your Physics - NEET / AIIMS KINEMATICS -1
 20  u  10  2
 20  u  20  u  0m / s Ans
Example 16.
A particle starts with initial velocity 2.5 m/s along the x direction and accelerates uniformly at the
rate 50 cm/s2. Find time taken to increase the velocity to 7.5 m/s.
2
u = 2.5 m/s a = 0.5 m/s
v = 7.5 m/s
Sol. v  u  at
t=0 t=?

7.5  2.5  0.5 t

5.0  0.5 t

50
t  10 sec
5
Example 17.
A particle starts with a constant acceleration. At a time t second speed is found to be 100 m/s and
one second later speed becomes 150 m/s. Find acceleration of the particle.
Sol. From equation (1) of motion v = u + at
 100 = 0 + at
100 = at ... (1)
Now consider velocity one second later -

v '  0  a  t  1

 150  a  t  1 ... (2)


On subtracting equation (1) from equation (2)

a  50 m / s2
Example 18.
A truck starts from rest with an acceleration of 1.5 ms-2 while a car 150 metre behind starts from
rest with an acceleration of 2 ms-2. (a) How long will it take before both the truck and car are side
by side and (b) How much distance is travelled by each.

1 2
Sol. (a) sT  at
2

1
sT  1.5  t 2 .... (1)
2
Distance covered by car when car one overtakes the truck

Page # 29 Tarun Mankad (M.Tech. - IIT Kanpur)


Master Your Physics - NEET / AIIMS KINEMATICS -1
1
sc   2 t2
2

1
 sT  150    2 t 2 .... (2)
2

sT  150 2 150 20 4
Divide equation (2) by equation (1) sT

1.5        1  s  15  3
T

150 4 1
  1 or sT  450
sT 3 3

Distance travelled by car = 450 + 150 = 600 metre

1 2
(b) Now by equation (1) sT  at
2

1
450   1.5  t 2
2

450  2
t2   t  300  2  24.5 sec
1.5
Therefore car will overtake the truck after 24.5 second.
Example 19.
A body travels a distance of 20 m in the 7th second and 24 m in 9th second. How much distance
shall it travel in the 15th second?
Sol. Here, s7 = 20 m ; s9 = 24 m,
s15 = ?
Let u and a be the initial velocity and uniform acceleration of the body.

a
We know that, sn  u   2n  1
2

a
 s7  u   2  7  1
2

13a
or 20  u  .... (i)
2

a
and s9  u  (2  9  1)
2

7
or 24 = u + a ....... (ii)
2

Subtracting (ii) form (i), we get


4 = 2a or a = 2 ms–2
Putting this value in (i), we get

Page # 30 Tarun Mankad (M.Tech. - IIT Kanpur)


Master Your Physics - NEET / AIIMS KINEMATICS -1
13
20  u  2 or 20 = u + 13
2
or u = 20 – 13 = 7 ms–1

a 2
Hence, s15 = u  (2 × 15 – 1) = 7 + × 29 s15 = 36 m Ans.
2 2
Example 20.
A person travelling at 43.2 km/h applies the brakes giving a deceleration of 6 m/s2 to his scooter.
How far will it travel before stopping?

5
Sol. Here, u  43.2km / h = 43.2  m/s
18
Deceleration; a = 6 m/s2 v= 0 s=? Using v2 = u2 - 2as
0 = (12)2 – 2 x 6 s 144 = 2 x 6s

144
s  12m Ans.
12
Example 21.
A bullet going with speed 350 m/s enters in a concrete wall and penetrates a distance of 5 cm before
coming to rest. Find deceleration.
Sol. Here, u = 350 m/s, s = 5 cm, v = 0 m/s and a= ?

u = 350 m/s
By using v 2  u2  2as 5cm

0  u2  2as

u2
u  2as or
2 a
2s

350  350
a= = 12.25 × 105 m/sec2
2  0.05

Page # 31 Tarun Mankad (M.Tech. - IIT Kanpur)


Master Your Physics - NEET / AIIMS KINEMATICS -1

Practice Problem - 7
Theoretical Questions
pickpocket rides on the motorcycle of a wait-
1. A particle starts moving from position of rest ing friend when the jeep is at a distance d away,
under a constant acceleration. If it travels a and the motorcycle starts with a constant ac-
distance x in t second, what distance will it celeration a. Show that the pickpocket will be
travel in next t second ?
caught if v  2ad .
2. A car travelling at 72 km h–1 takes a U turn in
10 seconds. What is the acceleration of the car 11. An object having a velocity 4.0 m/s is acceler-
? ated at the rate of 1.2 m/s2 for 5.0 s. Find the
distance travelled during the period of accel-
3. A body starting from rest has an acceleration
eration.
of 20 ms–2. Calculate the distance travelled by
it in 6th second. 12. A person travelling at 43.2 km/h applies the
brake giving a deceleration of 6.0 m/s2 to his
4. A train was moving at a rate of 36 km h–1.
scooter. How far will it travel before
When the brakes were applied, it comes to rest
stopping ?
in a distance of 200 m. Calculate the retarda-
tion produced in the train. 13. A train starts from rest and moves with a con-
stant acceleration of 2.0 m/s2 for half a minute.
5. A body covers 12 m in 2nd second and 20 m in
The brakes are then applied and the train comes
4th second. Find what distance the body will
to rest in one minute. Find (a) the total dis-
cover in 4 seconds after the 5th second.
tance moved by the train, (b) the maximum
6. On turning a corner, a motorist rushing at 44 speed attained by the train and (c) the
ms–1 finds a child on the road 100 m ahead. He position(s) of the train at half the maximum
instantly stops the engine and applies the brakes speed.
so as to stop it within 1 m of the child. Calcu-
14. A particle starting from rest moves with con-
late time required to stop it.
stant acceleration. If it takes 5.0s to reach the
7. A body starting from rest, was observed to speed 18.0 km/h find (a) the average velocity
cover 20 m in 1 second and 40 m during the during this period and (b) the distance trav-
next second. How far had it travelled before elled by the particle during this period.
the first observation was taken ?
15. A police jeep is chasing a culprit going on a
8. An automobile starts from rest and acceler- motorbike. The motorbike crosses a turning at
ates uniformly for 30 second to a speed of 72 a speed of 72 km/h. The jeep follows it at a
km h–1. It then moves with a uniform velocity speed of 90 km/h, crossing the turning ten sec-
and it is finally brought to rest in 50 m with a onds later than the bike. Assuming that they
constant retardation. If the total distance trav- travel at constant speeds, how far from the
elled is 950 m, find the acceleration, the retar- turning will the jeep catch up with the bike ?
dation and total time taken.
Obective Questions
9. A particle starts from rest with a constant ac-
16. A particle experiences a constant acceleration
celeration. At a time t second, the speed is found
for 20 sec after starting from rest. If it travels
to be 100 m/s and one second later the speed
becomes 150 m/s. Find (a) the acceleration and a distance S1 in the first 10 sec and a distance
(b) the distance travelled during the (t + 1)th S2 in the next 10 sec, then
second.
(A) S1  S2 (B) S1  S2 / 3
10. A police inspector in a jeep in chasing a pick-
(C) S1  S2 / 2 (D) S1  S2 / 4
pocket on a straight road. The jeep is going at
its maximum speed  (assumed uniform). The

Page # 32 Tarun Mankad (M.Tech. - IIT Kanpur)


Master Your Physics - NEET / AIIMS KINEMATICS -1
17. A point moves with uniform acceleration and and B starts in the same direction with constant
v1 ,v 2 and v 3 denote the average velocities in acceleration of 4 m / s 2 , then B will catch A
the three successive intervals of time t 1 , t 2 and after how much time
t 3 . Which of the following relations is correct (A) 10 sec (B) 20 sec
(C) 30 sec (D) 35 sec
(A) (v1  v 2 ) : (v 2  v 3 )  (t 1  t 2 ) : (t 2  t 3 ) 22. The average velocity of a body moving with
(B) (v1  v 2 ) : (v 2  v 3 )  (t 1  t 2 ) : (t 2  t 3 ) uniform acceleration travelling a distance of 3.06
m is 0.34 ms–1. If the change in velocity of the
(C) (v1  v 2 ) : (v 2  v 3 )  (t 1  t 2 ) : (t 1  t 3 ) body is 0.18ms–1 during this time, its uniform
(D) (v1  v 2 ) : (v 2  v 3 )  (t 1  t 2 ) : (t 2  t 3 ) acceleration is

18. The initial velocity of the particle is 10 m / sec (A) 0.01 ms–2 (B) 0.02 ms–2

and its retardation is 2m / sec 2 . The distance (C) 0.03 ms–2 (D) 0.04 ms–2
moved by the particle in 5th second of its 23. A particle starts from rest, accelerates at 2 m/
motion is s2 for 10s and then goes for constant speed
for 30s and then decelerates at 4 m/s2 till it
(A) 1 m (B) 19 m (C) 50 m (D) 75 m
stops. What is the distance travelled by it
19. The velocity of a body moving with a uniform
(A) 750 m (B) 800 m
acceleration of 2 m. / sec 2 is 10 m / sec . Its
(C) 700 m (D) 850 m
velocity after an interval of 4 sec is
24. A car, starting from rest, accelerates at the
(A) 12 m / sec (B) 14 m / sec rate f through a distance S, then continues at
(C) 16 m / sec (D) 18 m / sec constant speed for time t and then decelerates
f
20. Two cars A and B are travelling in the same at the rate to come to rest. If the total
2
direction with velocities v1 and v 2 (v1  v 2 ) .
distance traversed is 15 S, then
When the car is at a distance ahead of the car
, the driver of the car applied the brake 1 2 1 2
(A) S  ft (B) S  ft
producing a uniform retardation There will 2 4

be no collision when
1 2 1 2
(C) S  ft (D) S  ft
72 6
(v1  v 2 ) 2 v12  v22
(A) d  (B) d  25. A man is 45 m behind the bus when the bus
2a 2a
start accelerating from rest with acceleration
(v1  v 2 ) 2 v12  v 22 2.5 m/s2. With what minimum velocity should
(C) d  (D) d  the man start running to catch the bus ?
2a 2a
(A) 12 m/s (B) 14 m/s
21. Two cars A and B at rest at same point initially.
If A starts with uniform velocity of 40 m/sec (C) 15 m/s (D) 16 m/s

Page # 33 Tarun Mankad (M.Tech. - IIT Kanpur)


Master Your Physics - NEET / AIIMS KINEMATICS -1
11. MOTION UNDER GRAVITY
The most important example of motion in a straight line with constant acceleration is motion under
gravity. In case of motion under gravity.
(1) The acceleration is constant, i.e., a = g = 9.8 m/s2 and directed vertically downwards.
(2) The motion is in vacuum, i.e., viscous force or thrust of the medium has no effect on the motion.
[1] Body falling freely under gravity :
Taking initial position as origin and downward direction of motion as positive, we have
u=0 [as body starts from rest]
a=–g [as acceleration is in the downward direction]
So, if the body acquires velocity v after falling a distance h in time t, equations of motion, viz.

1
v = u + at; s  ut  at 2 and v 2  u2  2as
2

1 2
reduces to v = –gt ... (1) h  gt ..... (2) and v 2  2gh ..... (3)
2
These equations can be used to solve most of the problems of freely falling bodies as if.

t is given h is given v is given


From eqns. (1) and (2) From eqns. (2) and (3) From Eqns. (3)
and (1)

2h v
v = –gt t t
g g

1 2 v 2
and h  gt v  2gh h
2 2g

(i) If the body is dropped from a height H, as in time t is has fallen a distance h from its initial position,

' 2 1
the height of the body from the ground will be h  H  h with h    gt
2
 

Page # 34 Tarun Mankad (M.Tech. - IIT Kanpur)


Master Your Physics - NEET / AIIMS KINEMATICS -1
 1 2
(ii) As h    2  gt , i.e., h  t 2 , distance fallen in time, t,2t,3t etc., will be in the ratio of 12 : 22 : 32,
 
i.e., square of integers.
(iii) The distance fallen in the nth sec,

1 2 1 2 1
hn  hn 1  g  n   g  n  1  g  2n  1
2 2 2
So distances fallen in 1st, 2nd, 3rd sec etc. will be in the ratio of 1 : 3 : 5 i.e., odd integers only.
[2] Body projected vertically up :
Taking initial position as origin and direction of motion (i.e., vertically up) as positive.
here we have v = 0 [at highest point velocity = 0]
a = – g[as acceleration is downwards while motion upwards]
If the body is projected with velocity u and reaches the highest point at a distance h above the ground
in time t, the equations of motion viz.,

1
v = u + at; s  ut  at 2 and v 2  u2  2as
2

1 2
reduces to 0 = u – gt h  ut  gt and 0 = u2 – 2gh
2
Substituting the value of u from first equation in second and rearranging these,
u = gt ... (1)

1 2
h gt ... (2)
2

and u2  2gh ... (3)


These equations can be used to solve most of the problems of bodies projected vertically up as if.

t is given h is given u is given

From eqns. (1) and (2) From eqns. (2) and (3) From Eqns. (3) and (1)

2h u
u = gt t t
g g

1 u2
and h  gt 2 v  2gh h
2 2g

(A) (B) (C)

Page # 35 Tarun Mankad (M.Tech. - IIT Kanpur)


Master Your Physics - NEET / AIIMS KINEMATICS -1
IMPORTANT POINTS
(1) In case of motion under gravity for a given body, mass, acceleration, and mechanical energy remain
constant while speed, velocity, momentum, kinetic energy and potential energy change.
(2) The motion is independent of the mass of the body, as in any equation of motion, mass is not involved.
This is why a heavy and lighter body when released from the same height, reach the ground simultaneously
and with same velocity.

i.e t  2h / g and v  2gh

However, momentum, kinetic energy or potential energy depend on the mass of the body (all  mass)
(3) As from equation (2) time taken to reach a height h,

tu   2h / g
Similarly, time taken to fall down through a distance h,

tD   2h / g

so t u  tD   2h / g
So in case of motion under gravity time taken to go up a height h is equal to the time taken to fall
down through the same height h.
(4) If a body is projectd vertically up and it reaches a height h, then

u  2gh
and if a body falls freely through a height h, then

v  2gh   u
So in case of motion under gravity, the speed with which a body is projected up is equal to the speed
with which it comes back to the point of projection.

Example 22.
A juggler throws balls into air. He throws one whenever the previous one is at its highest point.
How high do the balls rise if he throws n balls each sec. Acceleration due to gravity is g.
Sol. Since the juggler is throwing n balls each second and he throws second ball when the first ball is
at the highest point, so time taken by each ball to reach the highest point is t = 1/n
Taking vertical upward motion of ball up to the highest point, we have
u = 0, a = – g, t = 1/n, u = ?
As v = u + at
so 0 = 0 u + (–g) 1/n
or u = g/n
Also v2 = u2 + 2as,
so 0 = u2 – 2gh
i.e., h = (u2/2g) = g/(2n2)
Example 23.
A ball is projected vertically up with an initial speed of 20 m/s on a planet where acceleration due
to gravity is 10 m/s2.

Page # 36 Tarun Mankad (M.Tech. - IIT Kanpur)


Master Your Physics - NEET / AIIMS KINEMATICS -1
(a) How long does it takes to reach the highest point?
(b) How high does it rise above the point of projection?
(c) How long will it take for the ball to reach a point 10 m above the point of projection?
Sol. As here motion is vertically upwards,
a = –g and v = 0
(a) From 1st equation of motion, i.e., v = u + at,
0 = 20 – 10t
i.e. t = 2 sec. Ans.
2 2
(b) Using v = u + 2as
0 = (20)2 – 2 x 10 x h
i.e h = 20 m. Ans.
1 1
(c) Using s  ut  at 2 , 10 = 20t (– ) x 10 x t2
2 2
i.e. t2 – 4t + 2 = 0 or t  2 2 ,
i.e. t = 0.59 sec. or 3.41 sec.
i.e., there are two times, at which the ball passes through h = 10 m, once while going up and then
coming down.
Example 24.
A ball is thrown vertically upwards from a bridge with an initial velocity of 4.9 m/s. It strikes the
water after 2s. If acceleration due to gravity is 9.8 m/s2 (a) What is the height of the bridge? (b)
With which velocity does the ball strike the water?
Sol. Taking the point of projection as origin and downward direction as positive,
 1
(a) Using s  ut    at 2 we have
2

 1
h  4.9  2    9.8  22  9.8m (u is taken to be negative as it is upwards)
2
(b) Using v = u + at
v = –4.9 + 9.8 x 2 = 14.7 m/s
Example 25.
A body is released from a height and falls freely towards the earth. Exactly 1 sec later another body
is released. What is the distance between the two bodies after 2 sec the release of the second body,
if g = 9.8 m/s2.
1 2
Sol. The 2nd body falls for 2s, so h2  g  2 ... (1)
2
While 1st has fallen for 2 + 1 = 3 sec so
1 2
h1  g3 ... (2)
2
 Separation between two bodies after 2 sec the release of 2nd body,,

1
d  h1  h2 
2

g 3 2  22   4.9  5  24.5m

Example 26.
If a body travels half its total path in the last second of its fall from rest, find : (a) The time and
(b) height of its fall. Explain the physically unacceptable solution of the quadratic time equation. (g

Page # 37 Tarun Mankad (M.Tech. - IIT Kanpur)


Master Your Physics - NEET / AIIMS KINEMATICS -1
2
= 9.8 m/s )
Sol. If the body falls a height h in time t, then

1 2
h gt [ u = 0 as the body starts from rest] ... (1)
2
Now, as the distance covered in (t – 1) second is

1 2
h'  g  t  1
h’ ... (2)
2
So from Equations (1) and (2) distance travelled in the last second.

' 1 1 2
hh – hh’  gt 2  g  t  1
2 2

1
i.e., h –h h’h'  g  2t  1
2

h
 h'  
But according to given problem as (h h–h’)
2

2
11  1
i.e., 22 h   2  g  2t  1
   

 1 2 1
or  2  gt  g  2t  1 [as from equation (1) h    gt 2 ]
  2
or t 2  4t  2  0

Page # 38 Tarun Mankad (M.Tech. - IIT Kanpur)


Master Your Physics - NEET / AIIMS KINEMATICS -1

Practice Problem - 8
Theoretical Questions
9. From the top of a tower 100 m in height a ball
1. A stone is thrown vertically upwards from the
is dropped and at the same instant another ball
surface of earth. What is the direction of the is projecred vertically upwards from the
velocity and acceleration of the stone (a) on ground so that it just reaches the top of tower.
its way up (b) on its way down. At what heigh do the two balls pass one an-
2. A man standing on the edge of a cliff throws a other?
stone straight up with initial speed u and then 10. A body falling from rest was observed to fall
throws another stone stright down with same through 78.4 m in 2 seconds. Find how long
initial speed and from the same position. Find had it been falling before it was observed?
the ratio of the speeds, the stones would have 11. A stone is dropped from a balloon going up
attained when they hit ground at the base of with a uniform velocity of 5.0 m/s. If the bal-
the cliff. loon was 50 m high when the stone was
3. Two balls of different masses (one lighter and dropped, find its height when the stone hits
other heavier) are thrown vertically upwards the ground. Take g = 10 m/s2.
with the same speed. Which one will pass 12. A ball is thrown vertically upward with a speed
through the point of projection in their down- of 20 m/s. Draw a graph showing the velocity
ward direction with the greater speed? of the ball as a function of time as it goes up
4. A ball is dropped from the roof of a tower of and then comes back.
height h. The total distance covered by it in 13. A ball is projected vertically upward with a
the last second of its motion is equal to the speed of 50 m/s. Find (a) the maximum height,
distance covered by it in first three seconds. (b) the time to reach the maximum height, (c)
What is the value of h? (g = 10m/s2) the speed at half the maximum height. Take g
5. A ball is thrown vertically upwards with a ve- = 10 m/s
locity of 20 ms-1 from the top of a multi-storey 14. A stone is thrown vertically upward with a
building. The height of the point from where speed of 28 m/s. (a) Find the maximum height
the ball is thrown is25. 0 m from the ground. reached by the stone. (b) Find its velocity one
(a) How high the ball will rise? and (b) how second before it reaches the maximum height.
long will it be before the ball hits the ground? (c) Does the answer of part. (d) change if the
Take, g = 10 ms-2. initial speed is more than 28 m/s such as 40
6. A stone is dropped from the top of a cliff and is m/s or 80 m/s?
found to travel 44.1 m in the last second be- 15. A person sitting on the top of a tall building is
fore it reaches the ground. Find the height of dropping balls at regular intervals of one sec-
the cliff. ond. Find the positions of the 3rd, 4th and 5th
7. A ball thrown up is caught by the thrower after ball when the 6th ball is being dropped.
4 second. How high did it go and with what 16. A ball is dropped from a height. If it takes 0.200
velocity was it thrown? How far below its high- s to cross the last 6.00 m before hitting the
est point was in 3 second after start? Accel- ground, find the height from which it was
eration due to gravity is 9.8 ms–2.
dropped. Take g = 10 m/s2.
8. From top of a tower 200 m in height, a ball is
Objective Questions
dropped and at the same time another ball is
17. A stone falls from a balloon that is descending
projected vertically upwards from the ground at a uniform rate of 12 m / s . The displace-
with a velocity of 50 ms–1. Find when and ment of the stone from the point of release
where the two balls will meet. after 10 sec is

Page # 39 Tarun Mankad (M.Tech. - IIT Kanpur)


Master Your Physics - NEET / AIIMS KINEMATICS -1
(A) 490 m (B) 510 m (C) 610 m (D) 725 m 26. Water drops fall at regular intervals from a tap
18. A ball is dropped on the floor from a height of which is 5 m above the ground. The third drop
10 m. It rebounds to a height of 2.5 m. If the is leaving the tap at the instant the first drop
ball is in contact with the floor for 0.01 sec, touches the ground. How far above the ground
the average acceleration during contact is is the second drop at that instant
(A) 2100 m/sec2 downwards (A) 2.50 m (B) 3.75 m (C) 4.00 m (D)1.25 m
(B) 2100 m/sec2 upwards 27. A body starts to fall freely under gravity. The
(C) 1400 m/sec2 (D) 700 m/sec2 distances covered by it in first, second and third
19. A body A is projected upwards with a velocity second are in ratio
of 98 m/s The second body B is projected (A) 1 : 3 : 5 (B) 1 : 2 : 3 (C) 1 : 4 : 9 (D) 1 : 5 : 6
upwards with the same initial velocity but after 28. A body is projected up with a speed ' u' and
4 sec. Both the bodies will meet after the time taken by it is T to reach the maximum
(A) 6 sec (B) 8 sec height H . Pick out the correct statement
(C) 10 sec (D) 12 sec (A) It reaches H / 2 in T / 2 sec
20. A body falls freely from rest. It covers as much (B) It acquires velocity u / 2 in T / 2 sec
distance in the last second of its motion as (C) Its velocity is at
covered in the first three seconds. The body (D) Same velocity at
has fallen for a time of 29. A body is thrown vertically upwards with a
(A) 3 s (B) 5 s (C) 7 s (D) 9 s velocity u . Find the true statement from the
21. A stone is dropped into water from a bridge following
44.1 m above the water. Another stone is (A) Both velocity and acceleration are zero at
thrown vertically downward 1 sec later. Both its highest point
strike the water simultaneously. What was the (B) Velocity is maximum and acceleration is
initial speed of the second stone zero at the highest point
(A) 12.25 m / s (B) 14.75 m / s (C) Velocity is maximum and acceleration is
(C) 16.23 m / s (D) 17.15 m / s g downwards at its highest point
22. A body is released from a great height and falls (D) Velocity is zero at the highest point and
freely towards the earth. Another body is maximum height reached is u2/2g
released from the same height exactly one 30. Two balls A and B of same masses are thrown
second later. The separation between the two from the top of the building. A, thrown upward
bodies, two seconds after the release of the with velocity V and B, thrown downward with
second body is velocity V, then
(A) 4.9 m (B) 9.8 m (C) 19.6 m (D) 24.5 m (A) Velocity of A is more than B at the ground
23. An object is projected upwards with a velocity (B) Velocity of B is more than A at the ground
of 100 m / s . It will strike the ground after (C) Both A & B strike the ground with same
(approximately) velocity
(A) 10 sec (B) 20 sec (D) None of these
(C) 15 sec (D) 5 sec 31. A parachutist after bailing out falls 50 m
24. A stone dropped from the top of the tower without friction. When parachute opens, it
touches the ground in 4 sec. The height of the decelerates at 2 m/s2. He reaches the ground
tower is about with a speed of 3 m/s. At what height, did he
(A) 80 m (B) 40 m (C) 20 m (D) 160 m bail out ?
25. A man in a balloon rising vertically with an (A) 293 m (B) 111 m (C) 91 m (D) 182 m
acceleration of 4.9 m / sec 2 releases a ball 2 32. When a ball is thrown up vertically with
sec after the balloon is let go from the ground. velocity Vo , it reaches a maximum height of
The greatest height above the ground reached ‘h’. If one wishes to triple the maximum height
by the ball is (g  9.8 m / sec 2 ) then the ball should be thrown with velocity
(A) 14.7 m (B) 19.6 m (C) 9.8 m (D) 24.5 m (A) 3 Vo (B) 3Vo (C) 9 Vo (D) 3 / 2Vo

Page # 40 Tarun Mankad (M.Tech. - IIT Kanpur)


Master Your Physics - NEET / AIIMS KINEMATICS -1
12. MOTION WITH NON-UNIFORM ACCELERATION (use of definite integrals)
tf
x =  v( t )dt (displacement in time interval t = ti to tf)
ti

The expression on the right hand side is called the definite integral of v(t) between
t = ti and t = tf. . Similarly change in velocity
tf
v = vf  vi =  a( t )dt
ti

Table : Some quantities defined as derivatives and integrals.

dx
v(t) = v = slope of xt graph
dt
dv
a (t) = a = slope of vt graphs
dt
dp
F (t) = F = slope of pt graph (p = linear momentum)
dt
tf
x =  dx =  v ( t ) dt x = area under vt graph
ti

tf
v =  dv =  a ( t ) dt v = area under at graph
ti

tf
p =  dp =  F ( t ) dt p = area under Ft graph
ti

xf
W=  dW =  F (x) dx W = area under Fx graph
xi

13. SOLVING PROBLEMS WHICH INVOLVES NON UNIFORM


ACCELERATION
13.1 Acceleration depending on velocity v or time t
v t
dv
By definition of acceleration, we have a =
dt
. If a is in terms of t, 
v0
dv =
 a ( t ) dt . If a is in terms
0

v t x
dv
of v,  a (v )
  dt
. On integrating, we get a relation between v and t, and then using  dx =
v0 0 x0

t
 v (t ) dt , x and t can also be related.
0

Page # 41 Tarun Mankad (M.Tech. - IIT Kanpur)


Master Your Physics - NEET / AIIMS KINEMATICS -1
13.2 Acceleration depending on velocity v or position x

dv dv dx dx dv dv
a=  a=  a= a=v
dt dx dt dt dx dx

This is another important expression for acceleration.

v x
If a is in terms of x,  v dv =  a ( x ) dx .
v0 x0

v x
v dv
If a is in terms of v, 
v0
a( v )
  dx
x0

x t
dx
On integrating, we get a relation between x and v. Using 
x0
dt
v ( x ) = 0 , we can relate x and t.

Example 27.
The displacement of a particle is given by y = a + bt + ct2 + dt4. Find the acceleration of a particle.

dy d
Sol. v
dt

dt
 
a  bt  ct 2  dt 4  b  2ct  4dt 3

dv
a  2c  12dt 2
dt
Example 28.
If the displacement of a particle is (2t2 + t + 5) meter then, what will be acceleration at t = 5 sec.

dx d dv d
Sol. v 
dt dt
 
2t 2  t  5  4t  1 m / s and a
dt

dt
 4t  1 a a=44 m/s
m /2s

Example 29.
The velocity of a particle moving in the x direction varies as V   x where  is a constant. Assuming
that at the moment t = 0 the particle was located at the point x = 0. Find the acceleration.

dv d 1 dx
Sol. a  x   . x 1/ 2 .
dt dt 2 dt

1 2
= . . x  a
2 x 2

Example 30.
The velocity of any particle is related with its displacement As; x  v  1 , Calculate acceleration
at x = 5 m.

Sol. x  v 1 x2  v  1 
v  x2  1 

Page # 42 Tarun Mankad (M.Tech. - IIT Kanpur)


Master Your Physics - NEET / AIIMS KINEMATICS -1
dv d 2 dx
Therefore a  
dt dt

x  1  2x
dt

 2x v  2x x 2  1  
at x = 5 m a  2  5  25  1  240m / s2

Example 31.

The velocity of a particle moving in the positive direction of x-axis varies as v=  x where 
is positive constant. Assuming that at the moment t = 0, the particle was located at x = 0 find, (i)
the time dependance of the velocity and the acceleration of the particle and (ii) the mean velocity
of the particle averaged over the time that the particle takes to cover first s metres of the path.

dx
Sol. (i) Given that v   x or  x
dt

dx x dx t
   dt or     dt
0 0
x x

Hence 
2 x   t or x   2 t 2 / 4 
dx 1 2
Velocity   t and
dt 2

d2 x 1 2
Acceleration  
dt 2 2

(ii) Time taken to cover first s metres

2 t 2 4s 2 s
s or t2  or t
4 2 

total dis tan ce s 1


v = or v  s 
total time 2 s 2

Example 32.
A particle moves in the plane xy with constant acceleration a directed along the negative y-axis. The
equation of motion of the particle has the form y = px – qx2 where p and q are positive constants.
Find the velocity of the partcle at the origin of coordinates.
Sol. Given that y = px – qx2

2
dy dx dx d2 y d2 x d2 x  dx 
 p  q.2x and 2
 p 2  2qx 2  2q  
dx dt dt dt dt dt  dt 

2
 dx 
or a  2q    2q2x
 dt 

d2 x d2 y
  0 (no acceleration along x-axis) and  a
dt 2 dt 2

Page # 43 Tarun Mankad (M.Tech. - IIT Kanpur)


Master Your Physics - NEET / AIIMS KINEMATICS -1

a a
 2x  or x 
2q 2q

 dy  dx
Further,  dt   p dt or y  p x
  x0

 a 
 y  p  
 2q 

 a ap2    
 a p2  1
Now   
2x  2y     or  
 2q
Ans.
 2q 2q  

SUMMARY
Rectilinear Motion : Rectilinear motion is motion, along a straight line or in one dimension.

Displacement : The vector joining the initial position of the particle to its final position during an
interval of time.

Distance : The length of the actual path travelled by a particle during a given time interval

displaceme nt xf  xi
Average Velocity = =
time int erval tf  ti

distance travelled
Average Speed =
time interval

 x  dx
Instantaneous Velocity : Vinst. = lim 
t  0 t
 =
  dt
Average Acceleration

change in velocity v f  vi
= time int erval = t t
f i

Instantaneous Acceleration :

dv  v 
a= = lim  
dt t 0  t 

Equations of Motion
(a) v = u + at
(b) s = ut + 1/2 at2
s = vt  1/2 at2
xf = xi + ut + 1/2 at2
(c) v2 = u2 + 2as

Page # 44 Tarun Mankad (M.Tech. - IIT Kanpur)


Master Your Physics - NEET / AIIMS KINEMATICS -1
(d) s = 1/2 (u + v) t
(e) sn = u + a/2 (2n  1)
Important Points to Remember
 For uniformly accelerated motion (a  0), xt graph is a parabola (opening upwards if a > 0 and
opening downwards if a < 0). The slope of tangent at any point of the parabola gives the velocity at that
instant.
 For uniformly accelerated motion (a  0), vt graph is a straight line whose slope gives the acceleration
of the particle.
 In general, the slope of tangent in xt graph is velocity and the slope of tangent in vt graph is the
acceleration.
 The area under at graph gives the change in velocity.
 The area between the vt graph gives the distance travelled by the particle, if we take all areas as
positive.
 Area under vt graph gives displacement, if areas below the taxis are taken negative.

Maxima and Minima


Conditions for maxima are:
dy d2 y
= 0 (b) <0
dx dx 2
Conditions for minima are:
dy d2 y
= 0(b) >0
dx dx 2

Motion with Non-Uniform Acceleration


tf tf
x =  v( t )dt v = vf  vi =  a( t )dt
ti ti

Solving Problems which Involves Nonuniform Acceleration


v t
If a is in terms of t,  dv =  a ( t ) dt
v0 0

v t
dv
If a is in terms of v, 
v0
a (v )
  dt
0

v x
If a is in terms of x,  v dv =  a ( x ) dx .
v0 x0

v x
v dv
If a is in terms of v, 
v0
a( v )
  dx
x0

Page # 45 Tarun Mankad (M.Tech. - IIT Kanpur)


Master Your Physics - NEET / AIIMS KINEMATICS -1

Practice Problem - 9
Theoretical Questions (A) 12 m/s (B) 18 m/s (C) 27 m/s(D) 36 m/s
11. A particle moves along a straight line such that
1. The displacement of a body as a function of
its displacement at any time t is given by
time is given by s = (3t2 + 4t + 7) m. Calculate
S = t3 – 6t2 + 3t + 4 metres The velocity when
the magnitude of its instantaneous velocity and the acceleration is zero is
acceleration at t = 1 s. (A) 3 ms-1 (B) –12 ms-1
2. The acceleration of a particle varies with time (C) 42 ms -1
(D) –9 ms-1
t-seconds according to the relation a=6t+6 m/ 12. The displacement is given by x = 2t2 + t + 5,
s2. Find the velocity and position as a function the acceleration at t = 2s is
of time. It is given that the particle starts from (A) 4 m/s2 (B) 8m/s2 (C) 10m/s2 (D) 15m/s2
13. The position x of a particle varies with time t
origin at t=0 with velocity 2 ms-1.
as x = at2 – bt3 . The acceleration of the
3. At t=0 a body is started from origin with some particle will be zero at time equal to
initial velocity. The displacement x(m) of the
a 2a a
body varies with time t(s) as x=-(2/3)t2+16t+2. (A) b (B) 3b (C) 3b (D)Zero
Find the initial velocity of the body and also 14. Equation of displacement for any particle is s
find how long does the body take to come to =(3t3+7t2+14t+8)m. Its acceleration at time t
rest? What is the acceleration of the body when = 1 sec is
it comes to rest? (A) 10 m/s2 (B) 16 m/s2
4. The position x of a particle varies with time t (C) 25 m/s2 (D) 32 m/s2
according to the relation x=t3 +3t2+2t. Find 15. The motion of a particle is described by the
equation x = a+bt2 where a = 15 cm and b =
velocity and acceleration as a function of time.
3 cm/s2. Its instantaneous velocity at time 3
5. The displacement of a particle along x-axis is sec will be
given by x=3+8t+7t2. Obtain its velocity and (A) 36 cm/sec (B) 18 cm/sec (C) 16
acceleration at t= 2s. cm/sec (D) 32 cm/sec
6. The acceleration a in ms-2 of a particle is given 16. A body is moving according to the equation
by a= 3t2+2t=2, where t is the time. If the par- x=at+bt2 – ct3 where x = displacement and
ticle starts out with a velocity v=2 ms-1 at t=0, a,b and c are constants. The acceleration of
then find the velocity at the end of 2 s. the body is
(A) a+ 2bt (B) 2b+6ct
7. The displacement x of a particle along the x-
(C) 2b-6ct (D) 3b-6ct2
a1 a 17. The displacement of a particle, moving in a
axis at time t is given by x= t  2 t 2. Find straight line, is given by s=2t2+2t+4 where s
2 3
is in metres and t in seconds. The acceleration
the acceleration of the particle. of the particle is
8. A particle moves along a straight line such that (A) 2 m/s2 (B) 4 m/s2
its displacement s at any time t is given by s= (C) 6 m/s 2
(D) 8 m/s2
t3-6t2+3t+4m, t being in seconds. Find the ve- 18. A particle moves along X-axis in such a way
locity of the particle when the acceleration is that its coordinate X varies with time t
zero. according to the equation x = (2 - 5t + 6t2) m.
Objective Questions The initial velocity of the particle is
(A) –5m/s (B) 6 m/s
9. The displacement of a particle is given by
(C) -3 m/s (D) 3 m/s
y  a  bt  ct 2  dt 4 . The initial velocity and 19. A particle moves along x-axis as
acceleration are respectively x = 4 (t - 2) + a ( t - 2 )2
(A) b, -4d (B)-b, 2c Which of the following is true ?
(C) b,2c (D)2c, -4d (A) The initial velocity of particle is 4
10. The acceleration ‘a’ in m/s2 of a particle is (B) The acceleration of particle is 2a
given by a=3t2 + 2t + 2 where t is the time. (C) The particle is at origin at t = 0
If the particle starts out with a velocity (D) None of these
u = 2m/s at t = 0, then the velocity at the end
of 2 second is
Page # 46 Tarun Mankad (M.Tech. - IIT Kanpur)
Master Your Physics - NEET / AIIMS KINEMATICS -1

Mock Test-1
Time : 3 hrs. M. M. : 70
(One Mark Problems ) 10. A jet airplane travelling at the speed of 500 kmh–
1
1. Write the expression for distance covered in nth ejects its products of combustion at the speed
second by a uniformly accelerated body. of 1500 kmh–1 relative to the jet plane. What is
the speed of the latter with respect to an ob-
2. The position–time (x–t) graphs for two children
server on the ground ?
A and B returning from their school O to their
homes P and Q respectively are shown in fig- 11. Adjoining figure shows the x–t plot of one di-
ure. Choose the correct entries in the brackets mensional motion of a particle. Is it correct to
below : say that the particle moves in straight line for t
< 0 ? If not, suggest a suitable physical context
for this graph.

(A) (A/B) lives closer to the school than (B/A).


(B) (A/B) starts from the school earlier than 12. A ball is dropped from a height of 90 m on a
(B/A). floor. At each collision with the floor, the ball
(C) (A/B) walks faster than (B/A). loses one-tenth of its speed. Plot the speed-time
(D) A and B reach home at the (same/different) time. graph of its motion between t = 0 to 12 s.
(E) (A/B) overtakes (B/A) on the road (once/twice). 13. Explain clearly, with examples, the distinction
3. Give an example of a body possessing zero ve- between :
locity and still accelerating. (A) magnitude of displacement (sometimes
4. What will happen to a hydrogen balloon released called distance) over an interval of time, and
on the moon ? the total length of path covered by a particle
over the same interval ;
5. Two masses in the ratio 1 : 2 are thrown verti-
cally up with the same speed. What is the effect (B) magnitude of average velocity over an in-
on the time by the mass ? terval of time, and the average speed over the
same interval. [Average speed of a particle over
6. Why is it not neccessary for a body following an interval of time is defined as the total path
another, to stop, to avoid collision ? length divided by the time interval]. Show in
7. If in case of a motion, displacement is directly both (A) and (B) that the second quantity is ei-
proportional to the square of time elapsed, what ther greater than or equal to the first. When is
do you think about its acceleration i.e., con- the equality sign true ? [For the sake of conve-
stant or variable ? Explain why ? nience, consider one-dimensional motion only].
8. Why does the earth impart the same accelera- 14. Two parallel rail tracks run north-south. Train
tion to all bodies ? A moves due north with a speed of 54 km h–1
(Two Marks Problems) and train B moves due south with a speed of 90
9. A drunkard walking in a narrow lane takes 5 km h–1. What is the relative velocity of B with
steps forward and 3 steps backward, followed respect to A in m s–1?
again by 5 steps forward and 3 steps backward, 15. Deduce the following relation v2 – u2 = 2as
and so on. Each step is 1 m long and requires 1 where symbols have their usual meaning.
s. Plot the x–t graph of his motion. Determine 16. If the length and time period of an oscillating
graphically and otherwise how long the drunk- pendulum have errors of 1% and 2% respec-
ard takes to fall in a pit 13 m away from the tively, what is the error in the estimate of g ?
start.
Page # 47 Tarun Mankad (M.Tech. - IIT Kanpur)
Master Your Physics - NEET / AIIMS KINEMATICS -1
17. Two straight lines drawn on the same displace- each interval.
ment – time graph make angles 30o and 60o with
time axis respectively in the figure. Which line
represents greater velocity ? What is the ratio
of the velocity of line A to the velocity of line B
?

22. State with reasons which of these following


graphs cannot possible represent one-dimen-
sional motion of a particle.
18. The driver of a truck travelling with a velocity
v suddenly notices a brick wall in front of him
at a distance d. Is it better for him to apply
brakes or to make a circular turn without appling
brakes in order to just avoid crashing into the
wall ? Why ?
(Three Marks Problems) (A) (B)
19. In which of the following examples of motion,
can the body be considered approximately a
point object:
(A) a railway carriage moving without jerks
between two stations.
(B) a monkey sitting on top of a man, cycling
smoothly on a circular track. (C) (D)
(C) a spinning cricket ball that turns sharply on 23. The distance x travelled by a body in a straight
hitting the ground. line is directly proportional to t2. Decide on the
(D) a tumbling beaker that has slipped off the type of motion associated. If x  t 3 what change
edge of a table. will you observe ?
20. Figure gives the x–t plot of a particle executing 24. A police van moving on a highway with a speed
one-dimensional simple harmonic motion. Give of 30 km/h fires a bullet at a thief’s car speed-
the signs of position, velocity and acceleration ing away in the same direction with a speed of
variables of the particle at t = 0.3 s, 1.2 s, –1.2
192 km/h. If the muzzle speed of the bullet is
s. 150 ms–1, with what speed does the bullet hit
the thief’s car ?
25. On a two lane road, car A is travelling with a
speed of 36 kmh–1. Two cars B and C approach
car A in opposite directions with a speed of 54
kmh–1 each. At a certain instant, when distance
AB is equal to AC, both being 1 km, B decides
to overtake A before C does. What minimum
acceleration of car B is required to avoid an
accident ?
26. A car moving along a straight highway with
21. Figure gives the x–t plot of a particle in one- speed of 126 km h–1 is brought to a stop within
dimensional motion. Three different equal in- a distance of 200 m. What is the retardation of
tervals of time are shown. In which interval is the car (assumed uniform) and how long does it
the average speed the greatest and in which is it take for the car to stop?
the least? Give the sign of average velocity for

Page # 48 Tarun Mankad (M.Tech. - IIT Kanpur)


Master Your Physics - NEET / AIIMS KINEMATICS -1
27. A man walks on a straight road from his home placement over a given time inteval.
to a market 2.5 km away with a speed of 5 km/
hr. Finding the market closed, he instantly turns, 1
(B) Establish the relation x  v0t  at 2
and walks back home with a speed of 7.5 km/ 2
hr. What is the (a) magnitude of average veloc- graphically.
ity, and (b) average speed of the man over the (C) A car moving with a speed of 126 km/h is
interval of time (i) 0–30 minutes (ii) 0–50 min- brought to a stop within a distance of 200 m.
utes (iii) 0–40 minutes ? Calculate the retardation of the car and the time
( Five Marks Questions) required to stop it.
28. Derive the three equations of motion by calcu- 30. Draw velocity–time graph of uniformly accel-
lus method. Express conditions under which they erated motion in one dimension. From the ve-
can be used. locity time graph of uniform accelerated mo-
29. (A) With the help of a simple case of an object tion deduce the equations of motion in distance
moving with constant velocity show that the area and time.
under velocity-time curve represents the dis-

Mock Test-2
Time : 3 hrs. M. M. : 70

(One Mark Problems) 11. Do the following two graphs represent same
1. What will be the nature of velocity–time graph type of motion ? Name the motion.
for a uniform motion ?
2. The position coordinates of a moving particle
is given by x = 6 + 18t + 9t2 (x in metres and t in
seconds). What is its velocity at t = 2 sec.?
3. A lorry and a car with the same kinetic energy
are brought to rest by the application of brakes
which provide equal retarding forces. Which of
them will come to rest in a shorter 12. A body starts from rest and moves along a
distance ? straight line. It has uniformly accelerated mo-
tion upto time t1 During the interval t2 - t1 it
4. Write two uses of v-t graph ?
moves with uniform velocity. After time t2 its
5. What is the ratio of SI to CGS unit of accelera- motion is retarded, and it comes to rest at time
tion ? t3. Draw the velocity-time graph.
6. What does speedometer of a car indicate ? 13. Which of the two : Velocity or acceleration
7. When a body accelerates by  t , what is the gives the direction motion of a body ? Justify
velocity after time ‘t’, when it starts from rest? your answer by an example.
8. What is the ratio of the time taken to go up and 14. A ball is thrown vertically up with a velocity
come down by a body thrown vertically up? of 20 m/s. Construct time-acceleration and
(Two marks problems ) time-displacement graph.
9. Two balls have the same size, but one is denser 15. Two balls of different masses are thrown verti-
than the other. Assume that the air resistance is cally upwards with same initial speed. Which
same in each case. Show that when they are one will rise to the greater height ? Which of
released simultaneously from the same height, the two will come back with greater speed to
the heavier ball will reach the ground first. the point of projection ?
10. Two bodies of different masses ml and m2 are 16. Derive the relation, v2 = u2 + 2as where the
dropped from two different heights ‘a’ and ‘b’. symbols carry usual meaning.
What is the ratio of time taken by the two to 17. A body dropped from a moving frame of refer-
drop through these distances ? ence, will have the velocity of the frame. Ex-
plain.

Page # 49 Tarun Mankad (M.Tech. - IIT Kanpur)


Master Your Physics - NEET / AIIMS KINEMATICS -1
18. Velocity-time graph of a moving object is shown (D) To what height does the ball rise and after how
below. What is the acceleration of the object ? long does the ball return to the player’s hands?
Also draw displacement-time graph for the (Take g = 9.8 m s–2 and neglect air resistance).
motion of the object. 24. Read each statement below carefully and state
with reasons and examples, if it is true or
false :
A particle in one-dimensional motion
(A) with zero speed at an instant may have non-
zero acceleration at that instant
(Three Marks Problems) (B) with zero speed may have non-zero veloc-
19. Two cars A and B are running at velocities of ity,
60 km/hr and 45 km/hr respectively. Calculate (C) with constant speed must have zero accel-
the relative velocity of car A if: eration,
(i) they are both travelling eastwards and (D) with positive value of acceleration must be
(it) car A is travelling eastwards and car B is speeding up.
travelling westwards. 25. Suggest a suitable physical situation for each
20. A particle starts from rest, and its acceleration of the following graphs :
(a) plotted against time (t) is shown below. Plot
the corresponding velocity (v) against time (t).
Also plot the corresponding displacement (s)
against time (t).

26. A three-wheeler starts from rest, accelerates


uniformly with 1 m s–1 on a straight road for 10
s, and then moves with uniform velocity. Plot
the distance covered by the vehicle during the
nth second (n = 1, 2, 3, ....) versus n. What do
you expect this plot to be during accelerated
21. A motorboat covers the distance between two motion : a straight line or a parabola ?
ports on the river in t1 = 8 hr and t2 = 12 hr 27. Two stones are thrown up simultaneously from
downstream and upstream respectively. What the edge of a cliff 200 m high with initial speeds
is the time required for the boat to cover this of 15 m s–1 and 30 m s–1. Verify whether the
distance in still water ? graph shown, correctly represents the time varia-
22. A body starts accelerating uniformly a from a tion of the relative position of the second stone
velocity ‘u’ and travels in a straight line. Prove with respect to the first. Neglect air resistance
and assume that the stones do not rebound after
a
that it covers a length of u  (2t  1) in the tth hitting the ground. Take g = 10 m s–2. Give the
2 equations for the linear and curved parts of the
second of motion. plot.
23. A player throws a ball upwards with an initial
speed of 29.4 ms–1.
(A) What is the direction of acceleration during the
upward motion of the ball ?
(B) What are the velocity and acceleration of the
ball at the highest point of its motion ?
(C) Choose the x = 0 m and t = 0 s to be the location
and time of the ball at its highest point, verti-
cally downward direction to be the positive di-
rection of x-axis, and give the signs of position,
velocity and acceleration of the ball during its
upward, and downward motion.

Page # 50 Tarun Mankad (M.Tech. - IIT Kanpur)


Master Your Physics - NEET / AIIMS KINEMATICS -1
(Three Long Problems)
28. A point object is thrown vertically upwards at
such a speed that it returns to the thrower after 6
seconds. With what speed was it thrown up and
how high did it rise? Plot speed time graph for
the object and use it to find the distance travelled
by it in the last second of its journey.
29. Derive an equation for the distance covered by a
uniformly accelerated body in nth second of its
motion. A body travels half its total path in the
last second of its fall from rest.
30. On a long horizontal moving belt, a child runs
to and for with a speed 9 km h–1 (with respect
to the belt) between his father and mother located
50 m apart on the moving belt. The belt moves
with a speed of 4 km h–1. For an observer on
a stationary platform outside, what is the
(A) speed of the child running in the direction of
motion of the belt ?
(B) speed of the child running opposite to the
direction of motion of the belt ?
(C) time taken by the child in (a) and (b) ?
Which of the answers alter if motion is viewed
by one of the parents ?

Page # 51 Tarun Mankad (M.Tech. - IIT Kanpur)


Master Your Physics - NEET / AIIMS KINEMATICS -1

Objective Questions
1. An athlete completes one round of a circular
(B) A body having zero velocity will neces-
track of radius R in 40 sec. What will be his
sarily have zero acceleration.
displacement at the end of 2 min. 20 sec.
(C) A body having uniform speed can have
(A) Zero (B) 2R
only uniform acceleration.
(C) 2R (D) 7R
(D) A body having non–uniform velocity will
2. A person travels along a straight road for half
have zero acceleration.
the distance with velocity 1 and the remain-
8. A particle experiences a constant acceleration
ing half distance with velocity 2. The aver-
for 20 sec after starting from rest. If it travels
age velocity is given by :
a distance S1 in the first 10 sec and a distance
22 S2 in the next 10 sec, then :
(A) 12 (B) (A) S1 = S2 (B) S1 = S2 / 3
12
(C) S1 = S2 / 2 (D) S1 = S2 / 4
1   2 2 1 2 9. An electron starting from rest has a velocity
(C) (D) that increases linearly with the time that is 
2 1  2
= kt, where k = 2m / sec2. The distance trav-
3. The displacement–time graph for two par- elled in the first 3 seconds will be :
ticles A and B are straight lines inclined at (A) 9 m (B) 16 m
angles of 30º and 60º with the time axis. The (C) 27 m (D) 36 m
ratio of velocities of VA : VB is :
10. The displacement of a body is given to be
(A) 1 : 2 (B) 1 : 3 proportional to the cube of time elapsed. The
magnitude of the acceleration of the body is :
(C) 3 :1 (D) 1 : 3 (A) Increasing with time
4. A car moves for half of its time at 80 km/h (B) Decreasing with time
and for rest half of time at 40 km/h. Total (C) Constant but not zero
distance covered is 60 km. What is the aver- (D) Zero
age speed of the car : 11. The acceleration of a moving body can be
(A) 60 km / h (B) 80 km / h found from :
(C) 120 km / h (D) 180 km / h (A) Area under velocity–time graph
5. The ratio of the numerical values of the av- (B) Area under distance–time graph
erage velocity and average speed of a body is (C) Slope of the velocity–time graph
always: (D) Slope of distance–time graph
(A) Unity (B) Unity or less 12. The initial velocity of the particle is 10 m/sec
(C) Unity or more (D) Less than unity and its retardation is 2m/sec2. The distance
6. th
If a car covers 2/5 of the total distance with moved by the particle in 5th second of its
1 speed and 3/5th distance with 2 then average motion is :
speed is : (A) 1 m (B) 19 m
1 1   2 (C) 50 m (D) 75 m
(A) 12 (B)
2 2 13. The velocity of a body depends on time ac-
cording to the equation u = 20 + 0.1 t2. The
2 1 2 51 2 body is undergoing:
(C) (D)
1  2 31  2  2 (A) Uniform acceleration
(B) Uniform retardation
7. The correct statement from the following is :
(C) Non–uniform acceleration
(A) A body having zero velocity will not
(D) Zero acceleration
necessarily have zero acceleration.

Page # 52 Tarun Mankad (M.Tech. - IIT Kanpur)


Master Your Physics - NEET / AIIMS KINEMATICS -1
14. A body starts from rest. What is the ratio of 20. Consider the acceleration, velocity and dis-
the distance travelled by the body during the placement of a tennis ball as it falls to the
4th and 3rd second : ground and bounces back. Direction of which
(A) 7/5 (B) 5/7 of these changes in the process.
(C) 7/3 (D) 3/7 (A) Velocity only
15. The acceleration 'a' in m/s2 of a particle is (B) Displacement and velocity
given by a = 3t2 + 2t + 2 where t is the time. (C) Acceleration, velocity and displacement
If the particle starts out with a velocity u = (D) Displacement and acceleration
2m /s at t = 0, then th velocity at the end of 21. A car moving with a speed of 50 km/hr, can
2 second is : be stopped by brakes after at least 6m. It the
(A) 12 m/s (B) 18 m/s same car is moving at a speed of 100 km/hr,
(C) 27 m/s (D) 36 m/s the minimum stopping distance is :
16. For a moving body at any instant of time : (A) 6 m (B) 12 m
(A) If the body is not moving, the acceleration (C) 18 m (D) 24 m
is necessarily zero 22. A student is standing at a distance of 50
(B) If the body is slowing, the retardation is metres from the bus. As soon as the bus
negative. begins its motion with an acceleration of 1ms–
2
(C) If the body is slowing, the distance is , the student starts running towards the bus
negative. with a uniform velocity u. Assuming the
(D) If displacement, velocity and acceleration motion to be along a straight road, the mini-
at that instant are known, we can find the mum value of u, so that the student is able
displacement at any given time in future. to catch the bus is :
17. Two cars A and B at rest at same point ini- (A) 5 ms–1 (B) 8 ms–1
tially. If A starts with uniform velocity of 40 (C) 10 ms–1 (D) 12 ms–1
m/sec and B starts in the same direction with 23. A car starts from rest and moves with uniform
constant acceleration of 4m/s2, then B will acceleration 'a' on a straight road from time
catch A after how much time : t = 0 to t = T. After that, a constant decel-
(A) 10 sec (B) 20 sec eration brings it to rest. In this process the
(C) 30 sec (D) 35 sec average speed of the car is:
18. A body travels for 15 sec starting from rest
aT 3aT
with constant acceleration. If it travels dis- (A) (B)
4 2
tance S1, S2 and S3 in the first five seconds,
second five seconds and next five seconds aT
respectively the relation between S1, S2 and (C) (D) aT
2
S3 is :
24. An object accelerates from rest to a velocity
(A) S1 = S2 = S3 (B) 5S1 = 3S2 = S3
27.5 m/s in 10 sec then find distance covered
1 1 1 1 by object in next 10 sec :
(C) S1 = S2 = S3 (D) S1 = S2 = S3
3 5 5 3 (A) 550 m (B) 137.5 m
19. The position of a particle moving along the (C) 412.5 m (D) 275 m
x-axis at certain times is given below : 25. Starting from rest, acceleration of a particle
t (s) 0 1 2 3 is a = 2(t – 1). The velocity of the particle
x (m) –2 0 6 16 at t = 5s is:
Which of the following describes the motion (A) 15 m/sec (B) 25 m/sec
correctly : (C) 5 m/sec (D) None of these
(A) Uniform, accelerated 26. A man is 45 m behind the bus when the bus
(B) Uniform, decelerated start accelerating from rest with acceleration
(C) Non–uniform, accelerated 2.5 m/s2. With what minimum velocity should
(D) There is not enough data for generalization the man start running to catch the bus?

Page # 53 Tarun Mankad (M.Tech. - IIT Kanpur)


Master Your Physics - NEET / AIIMS KINEMATICS -1
(A) 12 m/s (B) 14 m/s 33. With what velocity a ball be projected verti-
(C) 15 m/s (D) 16 m/s cally so that the distance covered by it in 5th
27. A body A is projected upwards with a velocity second is twice the distance it covers in its 6th
of 98 m/s. The second body B is projected second (g = 10 m/s2)
upwards with the same initial velocity but (A) 58.8 m/s (B) 49 m/s
after 4 sec. Both the bodies will meet after : (C) 65 m/s (D) 19.6 m/s
(A) 6 sec (B) 8 sec 34. A body thrown vertically upwards with an
(C) 10 sec (D) 12 sec initial velocity u reaches maximum height in
28. A body falls freely from rest. It covers as 6 seconds. The ratio of the distance travelled
much distance in the last second of its motion by the body in the first second and the seventh
as covered in the first three seconds. The body second is:
has fallen for a time of : (A) 1 : 1 (B) 11 : 1
(A) 3 s (B) 5 s (C) 1 : 2 (D) 1 : 11
(C) 7 s (D) 9 s 35. A body is thrown vertically upwards with a
29. An iron ball and a wooden ball of the same velocity u. Find the true statement from the
radius are released from the same height in following:
vacuum. They take the same time to reach the (A) Both velocity and acceleration are zero at
ground. The reason for this is : its highest point.
(A) Acceleration due to gravity in vacuum is (B) Velocity is maximum and acceleration is
same irrespective of the size and mass of the zero at the highest point.
body. (C) Velocity is maximum and acceleration is
(B) Acceleration due to gravity in vacuum g downwards at its highest point.
depends upon the mass of the body. (D) Velocity is zero at the highest point and
(C) There is no acceleration due to gravity in maximum height reached is u2/2g.
vacuum. 36. A particle when thrown, moves such that it passes
(D) In vacuum there is a resistance offered to from same height at 2 and 10 s, the height is :
the motion of the body and this resistance (A) g (B) 2g
depends upon the mass of the body. (C) 5g (D) 10 g
30. A stone thrown upward with a speed u from 37. Two balls A and B of same masses are thrown
the top of the tower reaches the ground with from the top of the building. A, thrown up-
a velocity 3u. The height of the tower is : ward with velocity V and B, thrown down-
(A) 3u2 / g (B) 4u2 / g ward with velocity V, then :
2
(C) 6u / g (D) 9u2 / g (A) Velocity of A is more than B at the ground
31. A man in a balloon rising vertically with an (B) Velocity of B is more than A at the ground
acceleration of 4.9 m/sec2 releases a ball 2 sec (C) Both A and B strike the ground with same
after the balloon is let go from the ground. velocity
The greatest height above the ground reaches (D) None of these
by the ball is (g = 9.8 m/sec2) 38. A body falling from a high Minaret travels 40
(A) 14.7 m (B) 19.6 m meters in the last 2 seconds of its fall to
(C) 9.8 m (D) 24.5 m ground. Height of Minaret in meters is
32. P, Q and R are three balloons ascending with (g = 10 m/s2)
velocities U, 4U and 8U respectively. If stones (A) 60 (B) 45
of the same mass be dropped from each, when (C) 80 (D) 50
they are at the same height, then : 39. A particle starts from rest. Its acceleration (A)
(A) They reach the ground at the same time (B) versus time (t) is as shown in the figure. The
Stone from P reaches the ground first maximum speed of the particle will be :
(C) Stone from R reaches the ground first
(D) Stone from Q reaches the ground first

Page # 54 Tarun Mankad (M.Tech. - IIT Kanpur)


Master Your Physics - NEET / AIIMS KINEMATICS -1

(A) 110 m/s (B) 55 m/s


(C) 550 m/s (D) 660 m/s
40. The variation of velocity of a particle with (A) Both the particles are having a uniformly
time moving along a straight line is illustrated accelerated motion.
in the following figure. The distance travelled (B) Both the particles are having a uniformly
by the particle in four seconds is : retarded motion.
(C) Particle (i) is having a uniformly accel-
erated motion while particle (ii) is having a
uniformly retarded motion.
(D) Particle (i) is having a uniformly retarded
motion while particle (ii) is having a uni-
formly accelerated motion.
(A) 60 m (B) 55 m
44. The graph of displacement /s time is,
(C) 25 m (D) 30 m
41. A ball is thrown vertically upwards. Which of
the following graph(s) represent velocity–time
graph of the ball during its flight (air resis-
tance is neglected) Its corresponding velocity–time graph will be :

(A) (B) (A) (B)

(C) (D) (C) (D)

42. The displacement versus time graph for a 45. Acceleration–time graph of a body is shown.
body moving in a straight line is shown in The corresponding velocity–time graph of the
figure. Which of the following regions repre- same body is :
sents the motion when no force is acting on
the body :

(A) ab (B) bc (A) (B)


(C) cd (D) de
43. Figure (i) and (ii) below show the displace-
ment–time graphs of two particles moving
along the x–axis. We can say that :
(C) (D)

Page # 55 Tarun Mankad (M.Tech. - IIT Kanpur)


Master Your Physics - NEET / AIIMS KINEMATICS -1
46. A particle moves along the sides AB, BC, CD 52. A particle is constrained to move on a straight
of a square of side 25m with a velocity of 15 line path. It returns to the starting point after
ms–1. Its average velocity is : 10 sec. The total distance covered by the
particle during this time is 30 m. Which of the
following statements about the motion of the
particle is false :
(A) Displacement of the particle is zero
(A) 15 ms–1 (B) 10 ms–1 (B) Average speed of the particle is 3 m/s
(C) 7.5 ms–1 (D) 5 ms–1 (C) Displacement of the particle is 30 m
47. A car A is travelling on a straight level road with (D) Both (A) and (B)
a uniform speed of 60 km/h. It is followed by 53. A particle moves along a semicircle of radius
another car B which is moving with a speed of 10m in 5 seconds. The average velocity of the
70 km/h. When the distance between them is 2.5 particle is:
km, the car B is given a deceleration of 20 km/ (A) 2 ms–1 (B) 4 ms–1
h2. After how much time will B catch up with (C) 2 ms–1 (D) 4 ms–1
A: 54. The instantaneous velocity of a body can be
(A) 1 hr (B) 1/2 hr measured :
(C) 1/4 hr (D) 1/8 hr (A) Graphically (B) Vectorially
48. A body moves 6 m north, 8 m east and 10 m (C) By speedometer (D) None of these
verticlly upwards, what is its resultant dis- 55. A body under the action of several forces will
placement from initial position : have zero acceleration :
(A) 10 2m (B) 10 m (A) When the body is very light
(B) When the body is very heavy
(C) 10/ 2m (D) 10 × 2 m (C) When the body is a point body
49. A person moves 30 m north and then 20 m (D) When the vector sum of all the forces
acting on it is zero
towards east and finally 30 2 m in south- 56. A motor car moving with a uniform speed of
west direction. The displacement of the per- 20 m/sec comes to stop on the application of
son from the origin will be : brakes after travelling a distance of 10 m. Its
(A) 10 m along north acceleration is :
(B) 10 m along south (A) 20 m / sec2 (B) –20 m / sec2
(C) 10 m along west (C) –40 m / sec2 (D) + 2m / sec2
(D) Zero 57. The velocity of a body moving with a uniform
50. A wheel of radius 1 metre rolls forward half acceleration of 2m / sec2 is 10 m/sec. Its
a revolution on a horizontal ground. The velocity after an interval of 4 sec is :
magnitude of the displacement of the point of (A) 12 m / sec (B) 14 m / sec
the wheel initially in contact with the ground (C) 16 m / sec (D) 18 m / sec
is : 58. A particle moving with a uniform acceleration
travels 24 m and 64 m in the first two con-
(A) 2 (B) 2 secutive intervals of 4 sec each. Its initial
velocity is :
(C) 2  4 (D)  (A) 1 m/sec (B) 10 m/sec
51. A car travels from A to B at a speed of 20 (C) 5 m/sec (D) 2 m/sec
km/hr and returns at a speed of 30 km/hr. The 59. An alpha particle enters a hollow tube of 4
average speed of the car for whole journey is m length with an initial speed of 1 km/s. It
: is accelerated in the tube and comes out it with
(A) 25 km/hr (B) 24 km/hr a speed of 9 km/s. The time for which it
(C) 50 km/hr (D) 5 km/hr remains inside the tube is:

Page # 56 Tarun Mankad (M.Tech. - IIT Kanpur)


Master Your Physics - NEET / AIIMS KINEMATICS -1
(A) 8 × 10–3s (B) 80 × 10–3
s 66. The displacement of a particle starting from
(C) 800 × 10 s –3 (D) 8 × 10 s–4 rest (at t = 0) is given by s = 6t2 – t3. The
60. Two cars A and B are travelling in the same time in seconds at which the particle will
direction with velocities 1 and 2 (1 > 2). attain zero velocity again, is :
When the car A is at a distance d ahead of (A) 2 (B) 4
the car B, the driver of the car A applied the (C) 6 (D) 8
brake producing a uniform retardation a. There 67. A particle travels 10m in first 5 sec and 10m
will be no collision when : in next 3 sec. Assuming constant acceleration
what is the distance travelled in next 2 sec.
( 1   2 ) 2  2  2 2
(A) d < (B) d < 1 (A) 8.3 m (B) 9.3 m
2a 2a
(C) 10.3 m (D) None of above
( 1   2 ) 2
1  2 2
2
68. The motion of a particle is described by the
(C) d > (D) d >
2a 2a equation u = at. The distance travelled by the
61. A body of mass 10 kg is moving with a constant
particle in the first 4 seconds :
velocity of 10 m/s. When a constant force acts
(A) 4a (B) 12a
for 4 seconds on it, it moves with a velocity (C) 6a (D) 8a
2 m/sec in the opposite direction. The accel-
69. A body of 5 kg is moving with a velocity of
eration produced in it is :
20 m/s. If a force of 100N is applied on it
(A) 3 m/sec2 (B) –3 m/sec2
for 10s in the same direction as its velocity,
(C) 0.3 m/sec2 (D) –0.3 m/sec2
what will now be the velocity of the body :
62. A car moving with a velocity of 10 m/s can be (A) 200 m/s (B) 220 m/s
stopped by the application of a constant force
(C) 240 m/s (D) 260 m/s
F in a distance of 20 m. If the velocity of the
70. The engine of a motorcycle can produce a
car is 30 m/s, it can be stopped by this force
maximum acceleration 5 m/s2. Its brakes can
in :
produce a maximum retardation 10 m/s2. What
20 is the minimum time in which it can cover a
(A) m (B) 20 m
3 distance of 1.5 km:
(C) 60 m (D) 180 m
(A) 30 sec (B) 15 sec
63. A boggy of uniformly moving train is sud-
(C) 10 sec (D) 5 sec
denly detached from train and stops after
71. A body A moves with a uniform acceleration
convering some distance. The distance cov-
a and zero initial velocity. Another body B,
ered by the boggy and distance covered by the
starts from the same point moves in the same
train in the same time has relation :
direction with a constant velocity . The two
(A) Both will be equal
(B) First will be half of second bodies meet after a time t. The value of t
(C) First will be 1/4 of second is :
(D) No definite ratio (A) 2/a (B) /a
64. A particle moves along a straight line such

that its displacement at any time t is given by (C) /2a (D)
S = t3 – 6t2 + 3t + 4 metres. The velocity when
2a
the acceleration is zero is : 72. The displacement of a particle is proportional
(A) 3 ms–1 (B) –12 ms–1 to the cube of time elapsed. How does the
(C) 42 ms–1 (D) –9 ms–1 acceleration of the particle depends on time
65. If a train travelling at 72 kmph is to be brought obtained :
to rest in a distance of 200 metres, then its (A) a  t2 (B) a  2t
retardation should be : (C) a  t 3 (D) a  t
(A) 20 ms–2 (B) 10 ms–2
(C) 2 ms–2 (D) 1 ms–2

Page # 57 Tarun Mankad (M.Tech. - IIT Kanpur)


Master Your Physics - NEET / AIIMS KINEMATICS -1
73. A body is moving with uniform acceleration 80. Two bodies of different masses ma and mb are
describes 40 m in the first 5 sec and 65 m in dropped from two different heights a and b.
next 5 sec. Its initial velocity will be : The ratio of the time taken by the two to cover
(A) 4 m/s (B) 2.5 m/s these distance are :
(C) 5.5 m/s (D) 11 m/s (A) a : b (B) b : a
74. Speed of two identical cars are u and 4u at (C) a : b (D) a2 : b2
a specific instant. The ratio of the respective 81. A stone is dropped into water from a bridge 44.1
distances in which the two cars are stopped m above the water. Another stone is thrown
from that instant is : vertically downward 1 sec later. Both strike the
(A) 1 : 1 (B) 1 : 4 water simultaneously. What was the initial speed
(C) 1 : 8 (D) 1 : 16 of the second stone:
75. A car, starting from rest, accelerates at the (A) 12.25 m/s (B) 14.75 m/s
rate f through a distance S, then continues at (C) 16.23 m/s (D) 17.15 m/s
constant speed for time t and then decelerates 82. A body is thrown vertically upwards. If air
at the rate f/2 to come to rest. If the total resistance is to be taken into account, then the
distance traversed is 15 S, then : time during which the body rises is :
1 2 1 2 (A) Equal to the time of fall
(A) S = ft (B) S = ft
2 4 (B) Less than the time of fall
1 2 1 2 (C) Greater than the time of fall
(C) S = ft (D) S = ft
72 6 (D) Twice the time of fall
76. A particle moves along x–axis as x = 4(t – 83. A body is released from a great height and
2) + a(t – 2)2. Which of the following is true? falls freely towards the earth. Another body
(A) The initial velocity of particle is 4. is released from the same height exactly one
(B) The acceleration of particle is 2a. second later. The separation between the two
(C) The particle is at origin at t = 0. bodies, two seconds after the release of the
(D) None of these second body is :
77. A body starting from rest moves with constant (A) 4.9 m (B) 9.8 m
acceleration. The ratio of distance covered by (C) 19.6 m (D) 24.5 m
the body during the 5th sec to that covered in 84. A body is released from the top of a tower of
5 sec is : height h. It takes t sec to reach the ground.
(A) 9/25 (B) 3/5 Where will be the ball after time t/2 sec :
(C) 25/9 (D) 1/25 (A) At h/2 from the ground
78. The distance between two particles is decreas- (B) At h/4 from the ground
ing at the rate of 6m/sec. If these particles (C) Depends upon mass and volume of the
travel with same speeds and in the same di- body
rection, then the separation increase at the rate (D) At 3h/4 from the ground
of 4m/sec. The particles have speeds as : 85. Two stones of different masses are dropped
(A) 5 m/sec; 1 m/sec simultaneously from the top of a building
(B) 4 m/sec ; 1 m/sec (A) Smaller stone hit the ground earlier
(C) 4 m/sec ; 2 m/sec (D) 5 m/ (B) Larger stone hit the ground earlier
sec ; 2 m/sec (C) Both stone reach the ground simultaneously
79. A stone falls from a balloon that is descending (D) Which of the stones reach the ground
at a uniform rate of 12 m/s. The displacement earlier depends on the composition of the stone
of the stone from the point of release after 10
86. A body thrown with an initial speed of 96 ft/
sec is :
sec reaches the ground after (g = 32 ft/sec)
(A) 490 m (B) 510 m
(A) 3 sec (B) 6 sec
(C) 610 m (D) 725 m
(C) 12 sec (D) 8 sec

Page # 58 Tarun Mankad (M.Tech. - IIT Kanpur)


Master Your Physics - NEET / AIIMS KINEMATICS -1
87. A stone is dropped from a certain height which (A) 5 m/sec (B) 10 m/sec
can reach the ground in 5 second. If the stone (C) 15 m/sec (D) 20 m/sec
is stopped after 3 second of its fall and then 95. A body is thrown vertically up from the
allowed to fall again, then the time taken by ground. It reaches a maximum height of 100m
the stone to reach the ground for the remain- in 5 sec. After what time it will reach the
ing distance is : ground from the maximum height position :
(A) 2 sec (B) 3 sec (A) 1.2 sec (B) 5 sec
(C) 4 sec (D) None of these (C) 10 sec (D) 25 sec
88. A particle is dropped under gravity from rest 96. A body thrown upwards with some velocity,
from a height h (g = 9.8 m/s2) and it travels reaches the maximum height of 20 m. An-
a distance 9h/25 in the last second, the height other body with double the mass thrown up,
h is : with double initial velocity will reach a maxi-
(A) 100 m (B) 122.5 m mum height of :
(C) 145 m (D) 167.5 m (A) 200 m (B) 16 m
89. A balloon is at a height of 81 m and is as- (C) 80 m (D) 40 m
cending upwards with a velocity of 12 m/s. 97. A balloon starts rising from the ground with
A body of 2 kg weight is dropped from it. If an acceleration of 1.25 m/s2 after 8s, a stone
g = 10 m/s2, the body will reach the surface is released from the balloon. The stone will
of the earth in : (g = 10 m/s2)
(A) 1.5 s (B) 4.025 s (A) Reach the ground is 4 second (B) Be-
(C) 5.4 s (D) 6.75 s gin to move down after being released
90. A ball is thrown vertically upwards from the (C) Have a displacement of 50 m ( D )
top of a tower at 4.9 ms–1. It strikes the pond Cover a distance of 40 m in reaching the
near the base of the tower after 3 seconds. The ground
height of the tower is : 98. A man throws a ball vertically upward and
(A) 73.5 m (B) 44.1 m it rises through 20 m and returns to his hands.
(C) 29.4 m (D) None of these What was the initial velocity (u) to the ball
91. A bullet is fired wtih a speed of 1000 m/sec and for how much time (T) it remained in the
in order to hit a target 100 m away. If g = air (g = 10 m/s2)
10 m/s2, the gun should be aimed : (A) u = 10 m/s, T = 2s
(A) Directly towards the target (B) u = 10 m/s, T = 4s
(B) 5 cm above the target (C) u = 20 m/s, T = 2s
(C) 10 cm above the target (D) u = 20 m/s, T = 4s
(D) 15 cm above the target 99. A ball is dropped from top of a tower of 100
92. A body starts to fall freely under gravity. The m height. Simultaneously another ball was
distance covered by it in first, second and thrown upward from bottom of the tower with
third second are in ratio : a speed of 50 m/s (g = 10 m/s2). They will
(A) 1 : 3 : 5 (B) 1 : 2 : 3 cross each other after:
(C) 1 : 4 : 9 (D) 1 : 5 : 6 (A) 1 s (B) 2 s
93. A stone is shot straight upward with a speed (C) 3 s (D) 4 s
of 20 m/sec from a tower 200 m high. The 100. A very large number of balls are thrown
speed with which it strikes the ground is vertically upwards in quick succession in such
approximately : a way that the next ball is thrown when the
(A) 60 m/sec (B) 65 m/sec previous one is at the maximum height. If the
(C) 70 m/sec (D) 75 m/sec maximum height is 5m, the number of ball
94. A body projected vertically upwards with a thrown per minute is (g = 10 m/s2)
velocity u returns to the starting point in 4 (A) 120 (B) 80
seconds. If g = 10 m/sec2, the value of u is : (C) 60 (D) 40

Page # 59 Tarun Mankad (M.Tech. - IIT Kanpur)


Master Your Physics - NEET / AIIMS KINEMATICS -1
101. A ball is released from the top of a tower of 107. A particle moving in a straight line covers half
height h meters. It takes T seconds to reach the distance with speed of 3 m/s. The other
the ground. What is the position of the ball half of the distance is covered in two equal
in T/3 seconds : time intervals with speed of 4.5 m/s and 7.5
(A) h/9 meters from the ground m/s respectively. The average speed of the
(B) 7h/9 meters from the ground particle during this motion is :
(C) 8h/9 meters from the ground (A) 4.0 m/s (B) 5.0 m/s
(D) 17h/18 meters from the ground (C) 5.5 m/s (D) 4.8 m/s
102. Two balls of same size but the density of one 108. A small block slides without friction down an
is greater than that of the other are dropped inclined plane starting from rest. Let Sn be the
from the same height, then which ball will distance travelled from time t = n – 1 to t =
reach the earth first (air resistance is negli-
Sn
gible) : n. Then is :
(A) Heavy ball S n 1
(B) Light ball
(C) Both simultaneously 2n  1 2n  1
(A) (B)
(D) Will depend upon the density of the balls 2n 2n  1
103. A packet is dropped from a balloon which is
2n  1 2n
going upwards with the velocity 12 m/s, the (C) (D)
veocity of the packet after 2 seconds will be :
2n  1 2n  1
(A) –12 m/s (B) 12 m/s 109. The  – t graph of a moving object is given
(C) –7.6 m/s (D) 7.6 m/s in figure. The maximum acceleration is
104. If a freely falling body travels in the last
second a distance equal to the distance trav-
elled by it in the first three second, the time
of the travel is :
(A) 6 sec (B) 5 sec
(C) 4 sec (D) 3 sec
105. From the top of a tower two stones, whose
masses are in the ratio 1 : 2 are thrown one (A) 1cm/sec2 (B) 2cm/sec2
2
straight up with an initial speed u and the (C) 3cm/sec (D) 6cm/sec2
second straight down with the same speed u. 110. The x–t graph shown in figure represents
Then, neglecting air resistance: (A) Constant velocity
(A) The heavier stone hits the ground with a (B) Velocity of the body is continuously chang-
higher speed ing
(B) The lighter stone hits the ground with a (C) Instantaneous velocity
higher speed (D) The body travels with constant speed upto
(C) Both the stones will have the same speed time t1 and then stops
when they hit the ground. 111. A lift is going up. The variation in the speed
(D) The speed can't be determined with the of the lift is as given in the graph. What is
given data. the height to which the lift takes the passen-
106. An object start sliding on a frictionless in- gers :
clined plane and from same height another
object start falling freely :
(A) Both will reach with same speed
(B) Both will reach with same acceleration
(C) Both will reach in same time
(D) None of above

Page # 60 Tarun Mankad (M.Tech. - IIT Kanpur)


Master Your Physics - NEET / AIIMS KINEMATICS -1
(A) 3.6 m (B) 28.8 m
(C) 36.0 m
(D) Cannot be calculated from the above graph (A) (B)
112. The velocity–time graph of a body moving in
a straight line is shown in the figure. The
displacement and distance travelled by the
body in 6 sec are respectively : (C) (D)

116. The area under acceleration–time graph gives :


(A) Distance travelled
(B) Change in acceleration
(C) Force acting
(D) Change in velocity
(A) 8 m, 16 m (B) 16 m, 8 m 117. Which graph represents the uniform accelera-
(C) 16 m, 16 m (D) 8 m, 8 m tion :
113. For the velocity–time graph shown in figure
below the distance covered by the body in last
two seconds of its motion is what fraction of (A) (B)
the total distance covered by it in all the seven
seconds.

(C) (D)

118. Which of the following velocity–time graphs


represent uniform motion :

(A) (B)
1 1
(A) (B)
2 4

1 2
(C) (D)
3 3 (C) (D)
114. The displacement–time graph of moving par-
ticle is shown below, 119. From the following displacement–time graph
find out the velocity of a moving body :

The instantaneous velocity of the particle is 1


(A) m/s (B) 3 m/s
negative at the point : 3
(A) D (B) F
(C) C (D) E 1
(C) 3 m/s (D) m/s
115. Which of the following graph represents 3
uniform motion :

Page # 61 Tarun Mankad (M.Tech. - IIT Kanpur)


Master Your Physics - NEET / AIIMS KINEMATICS -1
120. The  – t plot of a moving object is shown covering a distance S. When it covers an
in the figure. The average velocity of the object additional distance S, its speed would become :
during the first 10 seconds is : (A) 3u (B) 5u
(C) 11 u (D) 7u
127. Two trains one of length 100 m and another
of length 125 m, are moving in mutually
(A) 0 (B) 2.5 ms–1 opposite directions along parallel lines, meet
(C) 5 ms–1 (D) 2 ms–1 each other, each with speed 10 m/s. If their
121. A car travels a distance S on a straight road acceleration are 0.3 m/s2 and 0.2 m/s2 respec-
in two hours and then returns to the starting tively, then the time they take to pass each
point in the next three hours. Its average other will be :
velocity is : (A) 5 s (B) 10 s (C) 15 s (D) 20 s
(A) S / 5 (B) 2S / 5 128. A body starts from rest with uniform accel-
(C) S / 2 + S / 3 (D) None of the above eration. If its velocity after n second is , then
122. A body has speed V, 2V and 3V in first 1/3 its displacement in the last two seconds is :
of distance S, seconds 1/3 of S and third 1/ 2 ( n  1) ( n  1)
(A) (B)
3 of S respectively. Its average speed will be : n n
(A) V (B) 2V ( n  1) 2 ( n  1)
(C) (D)
(C) 18/11 V (D) 11/18 V n n
123. If the body covers one–third distance at speed 129. A particle is moving in a straight line and pass
1, next one third at speed 2 and last one through a point O with a velocity of 6 ms–1.
third at speed 3, then average speed will be : The particle moves with a constant retarda-
12   2 3  31 tion of 2 ms–2 for 4 s and there after moves
(A) with constant velocity. How long after leaving
1   2  3
O does the particle return to O :
1   2   3
(B) (A) 3 s (B) 8 s (C) Never (D) 4 s
3 130. A bird flies for 4 s with a velocity of |t – 2|
1 2  3 m/s in a straight line, where t is time in secnods.
(C)
12   2 3  31 It covers a distance of :
31 2 3 (A) 2 m (B) 4 m (C) 6 m (D) 8 m
(D)
12   2 3  31 131. A body is projected vertically up with a velocity
124. If the velocity of a particle is (10 + 2t2)m/s,  and after some time it returns to the point from
then the average acceleration of the particle which it was projected. The average velocity
between 2s and 5s is : and average speed of the body for the total time
(A) 2 m/s2 (B) 4 m/s2 of flight are :
(C) 12 m/s 2 (D) 14 m/s2 
(A)  /2 and /2 (B) 0 and /2
125. A thief is running away on a straight road in 
jeep moving with a speed of 9 ms–1. A police (C) 0 and 0 (D)  /2 and 0
man chases him on a motor cycle moving at 132. A stone is dropped from a height h. Simulta-
a speed of 10 ms–1. If the instantaneous sepa- neously, another stone is thrown up from the
ration of the jeep from the motorcycle is 100 ground which reaches a height 4 h. The two
m, how long will it take for the police to catch stones cross each other after time :
the thief:
h
(A) 1 s (B) 19 s (A) (B) 8gh
8g
(C) 90 s (D) 100 s
126. The speed of a body moving with uniform
acceleration is u. This speed is doubled while h
(C) 2gh (D)
2g
Page # 62 Tarun Mankad (M.Tech. - IIT Kanpur)
Master Your Physics - NEET / AIIMS KINEMATICS -1
133. Four marbles are dropped from the top of a
tower one after the other with an interval of
one second. The first one reaches the ground
after 4 seconds. When the first one reaches the
ground the distances between the first and
second, the second and third and the third and
forth will be respectively :
(A) 35, 25 and 15 m
(B) 30, 20 and 10 m
(C) 20, 10 and 5 m
(D) 40, 30 and 20 m
134. A balloon rises from rest with a constant
acceleration g/8. A stone is released from it
when it has risen to height h. The time taken
by the stone to reach the ground is :

h h
(A) 4 (B) 2
g g

2h g
(C) (D)
g h
135. Two bodies are thrown simultaneously from
a tower with same initial velocity 0 : one
vertically upwards, the other vertically down-
wards. The distance between the two bodies
after time t is:
1 2
(A) 20t + gt (B) 20t
2

1 2
(C) 0t + gt (D) 0t
2
136. A particle is projected upwards. The times
corresponding to height h while ascending and
while descending are t1 and t2 respectively.
The velocity of projection will be :
(A) gt1 (B) gt2
g( t 1  t 2 )
(C) g(t1 + t2) ((D)
2

Page # 63 Tarun Mankad (M.Tech. - IIT Kanpur)


Master Your Physics - NEET / AIIMS KINEMATICS -1

1. If a body A of mass M is thrown with velocity 6. Two trains travelling on the same track are
V at an angle of 30º to the horizontal and approaching each other with equal speeds of
another body B of the same mass is thrown 40 m/s. The drivers of the trains begin to
with the same speed at an angle of 60º to the decelerate simultaneously when they are just
horizontal. The ratio of horizontal range of A 2.0 km apart. Assuming the decelerations to
to B will be : [CBSE PMT 1992] be uniform and equal, the value of the decel-
(A) 1 : 3 (B) 1 : 1 eration to barely avoid collision should be:
[AMU 1995]
(C) 1 : 3 (D) 3 :1
(A) 11.8 m/s2 (B) 11.0 m/s2
2. A ball P is dropped vertically and another ball (C) 2.1 m/s2 (D) 0.8 m/s2
Q is thrown horizontally with the same veloci- 7. Two trains, each 50 m long are travelling in
ties from the same height and at the same opposite direction with velocity 10 m/s an
time. If air resistance is neglected, then 15 m/s. The time of crossing is :
[BHU 1994] [CPMT 1995]
(A) Ball P reaches the ground first (A) 2s (B) 4s
(B) Ball Q reaches the ground first (C) 2 3s (D) 4 3s
(C) Both reach the ground at the same time 8. An object is projected upwards with a velocity
(D) The respective masses of the two balls of 100 m/s. It will strike the ground after
will decide the time (approximately) [AFMC 1995]
3. A car accelerates from rest at a constant rate  (A) 10 sec (B) 20 sec
for some time, after which it deceleates at a (C) 15 sec (D) 5 sec
constant rate  and comes to rest. If the total 9. Water drops fall at regular intervals from a
time elapsed is t, then the maximum velocity tap which is 5 m above the ground. The third
acquired by the car is: [CPMT 1994] drop is leaving the tap at the instant the first
F  I t
2 2 F  I t
(B) G
2 2 drop touches the ground. How far above the
(A) G
H  JK H  JK ground is the second drop at that instant :
[CPMT 1995]
(   ) t t
(C) (D) (A) 2.50 m (B) 3.75 m
  
(C) 4.00 m (D) 1.25 m
4. A boy walks to his school at a distance of 6 km
10. The acceleration of a particle is increasing
with constant speed of 2.5 km/hr and walks
linearly with time t as bt. The particle starts
back with a constant speed of 4 km/hr. His
from the origin with an initial velocity 0. The
average speed for round trip expressed in km/
distance travelled by the particle in time t will
hr is: [AFMC 1995]
be: [CPMT 1995]
(A) 24/13 (B) 40/13
(C) 3 (D) 1/2 1 2 1 3
(A) 0t + bt (B) 0t + bt
5. The initial velocity of a particle is u (at t =
3 3
1 1
0) and the acceleration f is given by at. Which (C) 0t + bt3 (D) 0t + bt2
of the following relation is valid : 6 2
11. A particle reaches its highest point when it has
[BHU 1995]
covered exactly one half of its horizontal range.
t2 The corresponding point on the displacement
(A)  = u + at2 (B)  = u + a
2 time graph is characterised by :
(C)  = u + at (D)  = u [AIIMS 1995]

Page # 64 Tarun Mankad (M.Tech. - IIT Kanpur)


Master Your Physics - NEET / AIIMS KINEMATICS -1
(A) Negative slope and zero curvature 18. A truck and a car are moving with equal
(B) Zero slope and negative curvature velocity. On applying the brakes both will
(C) Zero slope and positive curvature stop after certain distance, then :[CPMT 1997]
(D) Positive slope and zero curvature (A) Truck will cover less distance before rest
12. A car travels the first half of a distance be- (B) Car will cover less distance before rest
tween two places at a speed of 30 km/hr and (C) Both will cover equal distance
the second half of the distance at 50 km/hr. (D) None
The average speed of the car for the whole 19. A ball of mass m1 and another ball of mass
journey is : [AFMC 1996] m2 are dropped from equal height. If time
(A) 42.5 km/hr (B) 40.0 km/hr taken by the balls are t1 and t2 respectively,
(C) 37.5 km/hr (D) 35.0 km/hr then : [BHU 1997]
13. A 120 m long train is moving in a direction (A) t1 = t2/2 (B) t1 = t2
with speed 20 m/s. A train B moving with 30 (C) t1 = 4t2 (D) t1 = t2/4
m/s in the opposite direction and 130 m long 20. A body sliding on a smooth inclined plane
crosses the first train in a time : requires 4 seconds to reach the bottom start-
[CPMT 1996] ing from rest at the top. How much time does
(A) 6 s (B) 36 s it take to cover one–fourth distance starting
(C) 38 s (D) None of these from rest at the top: [CPMT 1997]
14. A body dropped from a height h with an initial (A) 1 s (B) 2 s
speed zero, strikes the ground with a velocity (C) 4 s (D) 16 s
3 km/h. Another body of same mass is dropped 21. A stone dropped from a building of height h
from the same height h with an initial speed and it reaches after t seconds on earth. From
–u' = 4km/h. Find the final velocity of second the same building if two stones are thrown
body with which it strikes the ground : (one upwards and other downwards) with the
[CPMT 1996] same velocity u and they reach the earth surface
after t1 and t2 seconds respectively, then:
(A) 3 km/h (B) 4 km/h
[CPMT 1997]
(C) 5 km/h (D) 12 km/h
15. An aeroplane is flying horizontally with a t1  t 2
(A) t = t1 – t2 (B) t =
velocity of 600 km/h at a height of 1960 m. 2
When it is vertically at a point A on the ground, (C) t = t1t 2 (D) t = t12 t22
a bomb is released from it. The bomb strikes 22. A ball is dropped on the floor from a height
the ground at point B. The distance AB is : of 10 m. It rebounds to a height of 2.5 m. If
[CPMT 1996] the ball is in contact with the floor for 0.01
(A) 1200 m (B) 0.33 km sec, the average acceleration during contact is
(C) 3.33 km (D) 33 km : [CPMT 1997]
16. If a body starts from rest and travels 120 cm (A) 2100 m/sec2 downwards
in the 6th second, then what is the accelera- (B) 2100 m/sec2 upwards
tion: [AFMC 1997] (C) 1400 m/sec2
2 2
(A) 0.20 m/s (B) 0.027 m/s (D) 700 m/sec2
(C) 0.218 m/s2 (D) 0.03 m/s2 23. A stone dropped from the top of the tower
17. If a car at rest accelerates uniformly to a touches the ground in 4 sec. The height of the
speed of 144 km/h in 20 s. Then it covers a tower is about : [BHU 1998]
distance of [CPMT 1997] (A) 80 m (B) 40 m
(A) 20 m (B) 400 m (C) 20 m (D) 160 m
(C) 1440 m (D) 2880 m

Page # 65 Tarun Mankad (M.Tech. - IIT Kanpur)


Master Your Physics - NEET / AIIMS KINEMATICS -1
24. A ball is dropped downwards. After 1 second 31. Time taken by an object falling from rest to
another ball is dropped downwards from the cover the height of h1 and h2 is respectively
same point. What is the distance between them t1 and t2 then the ratio of t1 to t2 is :
after 3 seconds : [BHU 1998] [RPMT 1999]
(A) 25 m (B) 20 m
(C) 50 m (D) 9.8 m (A) h1 : h2 (B) h1 : h2
25. A train of 150 m length is going towards north (C) h1 : 2h2 (D) 2h1 : h2
direction at a speed of 10 m/sec. A parrot flies 32. A car moving with a speed of 40 km/h can
at the speed of 5m/sec towards south direction be stopped by applying brakes after atleast
parallel to the railway track. The time taken 2m. If the same car is moving with a speed
by the parrot to cross the train is : of 80 km/h, what is the minimum stopping
[BHU 1998] distance: [AFMC 2000]
(A) 12 sec (B) 8 sec (A) 8 m (B) 2 m
(C) 15 sec (D) 10 sec (C) 4 m (D) 6 m
26. The displacement x of a particle along a 33. The motion of a particle is described by the
straight line at time t is given by x = a0 + a1t equation x = a + bt2 where a = 15 cm and
+ a2t2. The acceleration of the particle is : b = 3 cm/s2. Its instantaneous velocity at time
[RPMT 1999] 3 sec will be : [AMU 2000]

(A) a0 (B) a1 (A) 36 cm/sec (B) 18 cm/sec


(B) 2a2 (D) a2 (C) 16 cm/sec (D) 32 cm/sec

27. The position x of a particle varies with time 34. A body is moving according to the equation
t as x = at2 – bt3. The acceleration of the x = at + bt2 – ct3 where x = displacement and
particle will be zero at time t equal to : a, b and c are constants. The acceleration of
the body is : [BHU 2000]
[BHU 1999]
(A) a + 2bt (B) 2b + 6ct
(A) a/b (B) 2a/3b
(C) 2b – 6ct (D) 3b – 6ct2
(C) a/3b (D) Zero
35. The distance travelled by a particle is propor-
28. A stone is thrown with an initial speed of 4.9
tional to the square of time, then the particle
m/s from a bridge in vertically upward direc-
travels with: [RPMT 2000]
tion. It falls down in water after 2 sec. The
(A) Uniform acceleration
height of the bridge is : [AFMC 1999]
(B) Uniform velocity
(A) 4.9 m (B) 9.8 m
(C) Increasing acceleration
(C) 19.8 m (D) 24.7 m
(D) Decreasing velocity
29. A body freely falling from the rest has a 36. Acceleration of a particle essentially changes
velocity ‘u’ after it falls through a height ‘h’. when : [RPMT 2000]
The distance it has to fall down for its velocity
(A) Direction of velocity changes
to become double, is : [NHU 1999]
(B) Magnitude of velocity changes
(A) 2 h (B) 4 h (C) Both of above
(C) 6 h (D) 8 h (D) Speed changes
30. Speed of a body on reaching the point from
37. The relation 3t = 3x + 6 describes the
which it was projected upwards, is :
[AIIMS 1999] displacement of a particle in one direction
where x is in metres and t in sec. The displace-
(A)  = 0 (B)  = 2u
ment, when velocity is zero, is :
(C)  = 0.5 u (D)  = u [CPMT 2000]
(A) 25 metres (B) 12 metres
(C) 5 metres (D) Zero
Page # 66 Tarun Mankad (M.Tech. - IIT Kanpur)
Master Your Physics - NEET / AIIMS KINEMATICS -1
38. The average velocity of a body moving with 45. One car moving on a straight road covers one
uniform acceleration travelling a distance of third of the distance with 20 km/hr and the
3.06 m is 0.34 ms–1. If the change in velocity rest with 60 km/hr. The average speed is :
of the body is 0.18 ms–1 during this time, its [AFMC 2002]
uniform acceleration is : [EAMCET 2000] (A) 40 km/hr (B) 80 km/hr
(A) 0.01 ms–2 (B) 0.02 ms–2 2
(C) 0.03 ms–2 (D) 0.04 ms–2 (C) 46 km/hr (D) 36 km/hr
3
39. Equation of displacement for any particle is 46. A man walks on a straight road from his home
s = 3t3 + 7t2 + 14t + 8m. Its acceleration at to a market 2.5 km away with a speed of 5
time t = 1 sec is : [CPMT 2000] km/h. Finding the market closed, he instantly
(A) 10 m/s2 (B) 16 m/s2 turns and walks back home with a speed of
(C) 25 m/s2 (D) 32 m/s2 7.5 km/h. The average speed of the man over
the interval of time 0 to 40 min. is equal
40. A stone is just released from the window of
to : [AMU 2002]
a train moving along a horizontal straight
track. The stone will hit the ground following : (A) 5 km / h (B) 25/4 km/h
(C) 30 / 4 km/h (D) 45 / 8 km/h
[BHU 2000]
47. A person travels along a straight road for the
(A) Straight path (B) Circular path
(C) Parabolic path (D) Hyperbolic path first half time with a velocity 1 and the next
half time with a velocity 2. The mean veloc-
41. A 150 m long train is moving with a uniform
ity V of the man is : [BHU 2002]
velocity of 45 km / h. The time taken by the
train to cross a bridge of length 850 metres 2 1 1 1   2
is : [CPMT 2001] (A)   (B) V =
V 1 2 2
(A) 56 sec (B) 68 sec
(C) 80 sec (D) 92 sec
1
42. The displacement of a particle, moving in a (C) V = 1 2 (D) V =
2
straight line, is given by s = 2t2 + 2t + 4 where
s is in metres and t in seconds. The accelera- 48. A particle starts from rest, accelerates at 2 m/
tion of the particle is : [CPMT 2001] s2 for 10 s and then goes for constant speed
(A) 2 m/s2 (B) 4 m/s2 for 30s and then decelerates at 4 m/s2 till it
(C) 6 m/s2 (D) 8 m/s2 stops. What is the distance travelled by it :
43. A body A starts from rest with an acceleration [AIIMS 2002]
a1. After 2 seconds, another body B starts (A) 750 m (B) 800 m
from rest with an acceleration a2. If they travel (C) 700 m (D) 850 m
equal distances in the 5th second, after the 49. Three different objects of masses m1, m2 and
start of A, then the ratio a1 : a2 is equal to m3 are allowed to fall from rest and from the
: [AIIMS 2001] same point ‘O’ along three different friction-
(A) 5 : 9 (B) 5 : 7 less paths. The speeds of the three objects, on
(C) 9 : 5 (D) 9 : 7 reaching the ground, will be in the ratio
44. The velocity of a bullet is reduced from 200 of : [AIIMS 2002]
m/s to 100 m/s while travelling through a (A) m1 : m2 : m3 (B) m1 : 2m2 : 3m3
wooden block of thickness 10cm. The retar-
1 1 1
dation, assuming it to be uniform, will be : (C) 1 : 1 : 1 (D) : :
m1 m2 m3
[AIIMS 2001]
(A) 10 × 104 m/s2
(B) 12 × 104 m/s2 50. From the top of a tower, a particle is thrown
(C) 13.5 × 104 m/s2 (D) 15 × 104 m/s2 vertically downwards with a velocity of 10 m/
s. The ratio of the distances, covered by it in

Page # 67 Tarun Mankad (M.Tech. - IIT Kanpur)


Master Your Physics - NEET / AIIMS KINEMATICS -1
the 3rd and 2nd seconds of the motion is (Take interval of 2 seconds. What should be the
g = 10 m/s2) [CPMT 2002] speed of the throw so that more than two balls
(A) 5 : 7 (B) 7 : 5 are in the sky at any time (Given g = 9.8 m/
(C) 3 : 6 (D) 6 : 3 s 2) : [CPMT 2003]
51. A particle (A) is dropped from a height and (A) At least 0.8 m/s
another particle (B) is thrown in horizontal (B) Any speed less than 19.6 m/s
direction with speed of 5 m/sec from the same (C) Only with speed 19.6 m/s
height. The correct statement is : (D) More than 19.6 m/s
[CBSE PMT 2002] 57. If a ball is thrwon vertically upwards with
(A) Both particles will reach at ground simul- speed u, the distance covered during the last
taneously t seconds of its ascent is : [CPMT 2003]
(B) Both particles will reach at ground with 1 2 1 2
same speed (A) gt (B) ut – gt
2 2
(C) Particle (A) will reach at ground first with
(C) (u – gt)t (D) ut d
respect to particle (B).
58. An aeroplane flies 400 m north and 300 m
(D) Particle (B) will reach at ground first with
west and then flies 1200 m upwards then net
respect to particle (A).
displacement is : [AFMC 2004]
52. A man goes 10 m towards North, then 20m
(A) 1200 m (B) 1300 m
towards east then displacement is :
(C) 1400 m (D) 1500 m
[AFMC 2003]
59. The numerical ratio of displacement to the
(A) 22.5 m (B) 25 m distance covered is always :[BHU 2004]
(C) 25.5 m (D) 30 m
(A) Less than one
53. The displacement of a particle is given by y (B) Equal to one
= a + bt + ct2 – dt4. The initial velocity and
(C) Equal to or less than one
acceleration are respectively :
(D) Equal to or greater than one
[CPMT 2003]
60. A 100 m long train is moving with a uniform
(A) b, –4d (B) –b, 2c velocity of 45 km/hr. The time taken by the
(C) b, 2c (D) 2c, –4d train to cross a bridge of length 1 km is :
54. Two balls are dropped from heights h and 2h [BHU 2004]
respectively from the earth surface. The ratio (A) 58 s (B) 68 s
of time of these balls to reach the earth is (C) 78 s (D) 88 s
[CPMT 2003] 61. A police jeep is chasing with, velocity of 45
km/h a thief in another jeep moving with
(A) 1 : 2 (B) 2 :1
velocity 153 km/h. Police fires a bullet with
(C) 2 : 1 (D) 1 : 4 muzzle velocity of 180 m/s. The velocity it
55. The acceleration due to gravity on the planet will strike the car of the thief is:
A is 9 times the acceleration due to gravity [CPMT 2004]
on planet B. A man jumps to a height of 2m (A) 150 m/s (B) 27 m/s
on the surface of A. What is the height of jump
(C) 450 m/s (D) 250 m/s
by the same person on the planet B :
62. A particle is thrown vertically upwards. If its
[CPMT 2003] velocity at half of the maximum height is 10
(A) 18 m (B) 6 m m/s, then maximum height attained by it is
(C) 2/3 m (D) 2/9 m (Take g = 10 m/s2) [CPMT 2004]
56. A man throws balls with the same speed (A) 8 m (B) 10 m
vertically upwards one after the other at an (C) 12 m (D) 16 m
Page # 68 Tarun Mankad (M.Tech. - IIT Kanpur)
Master Your Physics - NEET / AIIMS KINEMATICS -1
63. A body falls from a height h = 200m (at New
Delhi). The ratio of distance travelled in each
2 sec during t = 0 to t = 6 second of the (C) (D)
journey is : [CPMT 2004]
(A) 1 : 4 : 9 (B) 1 : 2 : 4
(C) 1 : 3 : 5 (D) 1 : 2 : 3 68. Two boys are standing at the ends A and B
64. A body is thrown vertically upwards with of a ground where AB = a. The boy at B starts
velocity u. The distance travelled by it in the running in a direction perpendicular to AB
fifth and the sixth seconds are equal. The with velocity 1. The boy at A starts running
velocity u is given by (g = 9.8 m/s2) : simultaneously with velocity  and catches
[BHU 2004] the other boy in a time t, where t is :
[CPMT 2005]
(A) 24.5 m/s (B) 49.0 m/s
(C) 73.5 m/s (D) 98.0 m/s
(A) a/ 2  12 (B) a 2 / 2  12
65. A man drops a ball downside from the roof
of a tower of height 400 meters. At the same (C) a / ( – 1) (D) a / ( + 1)
time another ball is thrown upside with a 69. What determines the nature of the path fol-
velocity 50 meter/sec from the surface of the lowed by the particle : [AFMC 2005]
tower, then they will meet at which height (A) Speed (B) Velocity
from the surface of the tower : (C) Acceleration (D) None of these
[CPMT 2003] 70. When a ball is thrown up vertically with
(A) 100 meters (B) 320 meters velocity V0, it reaches a maximum height of
(C) 80 meters (D) 240 meters 'h'. If one wishes to triple the maximum height
66. Which of the following velocity–time graph then the ball should be thrown with velocity.
shows a realistic situation for a body [AIIMS 2005]
in motion : [AIIMS 2004] (A) 3V0 (B) 3V0
(C) 9V0 (D) 3/2V0
71. An object is projected at an angle of 45º with
(A) (B) the horizontal. The horizontal range and the
maximum height reached will be in the ratio.
[AFMC 2005]
(A) 1 : 2 (B) 2 : 1
(C) (D) (C) 1 : 4 (D) 4 : 1

67. An object is moving with a uniform accelera-


tion which is parallel to its instantaneous di-
rection of motion. The displacement(s) –
velocity() graph of this object is :
[AIIMS 2003]

(A) (B)

Page # 69 Tarun Mankad (M.Tech. - IIT Kanpur)


Master Your Physics - NEET / AIIMS KINEMATICS -1

3 Assertion & Reason


Assertion & Reason Reason : Velocity is a vector quantity
Read the assertion and reason carefully to and speed is a scalar quantity.
mark the correct option out of the options
5. Assertion : An object can have constant
given below.
speed but variable velocity.
(A) If both assertion and reason are true
Reason : Speed is a scalar but velocity
and the reason is the correct explanation
is a vector quantity.
of the assertion.
6. Assertion : The speed of a body can be
(B) If both assertion and reason are true
negative.
but reason is not the correct explanation
of the assertion. Reason : If the body is moving in the
opposite direction of positive
(C) If assertion is true but reason is false.
motion, then its speed is
(D) If the assertion and reason both are negative.
false.
7. Assertion : The position–time graph of a
(e) If assertion is false but reason is true. uniform motion in one
1. Assertion : A body can have acceleration dimension of a body can have
even if its velocity is zero at negative slope.
a given instant of time. Reason : When the speed of body
Reason : A body is momentarily at rest decreases with time, the
when it reverses its direction position–time graph of the
of motion. moving body has negative
slope.
2. Assertion : Two balls of different masses
are thrown vertically upward 8. Assertion : A positive acceleration of a
with same speed. They will body can be associated with
pass through their point of a ‘slowing down’ of the body.
projection in the downward Reason : Acceleration is a vector
direction with the same speed. quantity.
Reason : The maximum height and 9. Assertion : A negative accelertion of a
downward velocity attained body can be associated with
at the point of projection are a ‘speeding up’ of the body.
independent of the mass of
Reason : Increase in speed of a moving
the ball.
body is independent of its
3. Assertion : If the displacement of the direction of motion.
body is zero, the distance
10. Assertion : When a body is subjected to
covered by it may not be zero.
a uniform acceleration, it
Reason : Displacement is a vector always move in a straight line.
quantity and distance is a
Reason : Straight line motion is the
scalar quantity.
natural tendency of the body.
4. Assertion : The average velocity of the
11. Assertion : Rocket in flight is not an
object over an interval of time
illustration of projectile.
is either smaller than or equal
to the average speed of the Reason : Rocket takes flight due to
object over the same interval. combustion of fuel and does

Page # 70 Tarun Mankad (M.Tech. - IIT Kanpur)


Master Your Physics - NEET / AIIMS KINEMATICS -1
not move under the gravity Reason : The relative velocity of a body
effect alone. with respect to itself is zero.
12. Assertion : The average speed of a body 18. Assertion : Displacement of a body may
over a given interval of time be zero when distance
is equal to the average travelled by it is not zero.
velocity of the body in the Reason : The displacement is the
same interval of time if a body longest distance between
moves in a straight line in initial and final position.
one direction.
19. Assertion : The equation of motion can
Reason : Because in this case distance be applied only if acceleration
travelled by a body is equal is along the direction of
to the displacement of the velocity and is constant.
body.
Reason : If the acceleration of a body
13. Assertion : Position–time graph of a is constant then its motion is
stationary object is a straight known as uniform motion.
line parallel to time axis.
20. Assertion : A bus moving due north takes
Reason : For a stationary object, a turn and starts moving
position does not change with towards east with same speed.
time. There will be no change in
14. Assertion : The slope of displacement– the velocity of bus.
time graph of a body moving Reason : Velocity is a vector–quantity.
with high velocity is steeper
21. Assertion : The relative velocity between
than the slope of
any two bodies moving in
displacement–time graph of
opposite direction is equal to
a body with low velocity.
sum of the velocities of two
Reason : Slope of displacement–time bodies.
graph = Velocity of the body.
Reason : Sometimes relative velocity
15. Assertion : Distance–time graph of the between two bodies is equal to
motion of a body having difference in velocities of the
uniformly accelerated motion two.
is a straight line inclined to
22. Assertion : The displacement–time graph
the time axis.
of a body moving with
Reason : Distance travelled by a body uniform acceleration is a
having uniformly accelerated straight line.
motion is directly
Reason : The displacement is
proportional to the square of
proportional to time for
the time taken.
uniformly accelerated motion.
16. Assertion : A body having non–zero
acceleration can have a 23. Assertion : Velocity–time graph for an
constant velocity. object in uniform motion
Reason : Acceleration is the rate of along a straight path is a
change of velocity. straight line parallel to the
17. Assertion : A body, whatever its motion time axis.
is always at rest in a frame Reason : In uniform motion of an
of reference which is fixed to object velocity increases as
the body itself. the square of time elapsed.

Page # 71 Tarun Mankad (M.Tech. - IIT Kanpur)


Master Your Physics - NEET / AIIMS KINEMATICS -1
24. Assertion : A body may be accelerated
even when it is moving
uniformly.
Reason : When direction of motion of
the body is changing then
body may have acceleration.
25. Assertion : A body falling freely may do
so with constant velocity.
Reason : The body falls freely, when
acceleration of a body is equal
to acceleration due to gravity.
26. Assertion : Displacement of a body is
vector sum of the area under
velocity–time graph.
Reason : Displacement is a vector
quantity.
27. Assertion : The position–time graph of a
body moving uniformly is a
straight line parallel to
position axis.
Reason : The slope of position–time
graph in a uniform motion
gives the velocity of an object.
28. Assertion : The average speed of an
object may be equal to
arithmetic mean of individual
speed.
Reason : Average speed is equal to total
distance travelled per total
time taken.
29. Assertion : The average and
instantaneous velocities have
same value in a uniform
motion.
Reason : In uniform motion, the
velocity of an object increases
uniformly.
30. Assertion : The speedometer of an
automobile measure the
average speed of the
automobile.
Reason : Average velocity is equal to
total displacement per total
time taken.

Page # 72 Tarun Mankad (M.Tech. - IIT Kanpur)


Master Your Physics - NEET / AIIMS KINEMATICS -1
Answer Key
Practice Problem-1
5. b and c 6. (A) 7. (A) 8. (C) 9. (A)

Practice Problem-2
6. (A) 10.4 km (B) 0.62 h. (C) 16.8 km/h  17 km./h
7. (A) all tie; (B) 4, tie of 1 and 2, then 3
8. (A) +40km/h;(B) 40 km/h
9. (A) 1.74 m/s (B) (2. 135 m/s)
10. (B) 11. (B) 12. (C) 13. (B) 14. (A)
15. (B) 16. (C) 17. (C) 18. (D) 19. (C)
20. (B)

Practice Problem-3
8. (a) plus; (b) minus; (c) minus; (d) plus
9. 20 m/s2, in the direction opposite to its initial velocity
10. 3600 km/ hr, 3600 km/ hr
11. (A) 12. (D) 13. (C)

Practice Problem-4
1
1.  150o 2.  3.  1 5. (i) x = 3 (ii) x = –2
6
6. (i) y = 5 (ii) y = –a 7. (i) y = –3 (ii) x = 2
3 6
8. (a)  (b) 0 (c) (d) 3
2 0

9. 3 10. 135o

1
11. ,104.5 Crores 12. y = 0 and x = 0 13. x – 2y + 10 = 0
2
14. y = mx 15. 30o and 150o 16. (C) 17. (A)

Practice Problem-5
1. () 2. () 3. ()
4. A straight line parallel to time axis.
5. A straight line inclined to time axis.
6. The uniform velocity of the object.
7. Fig. (a) Infinity (b) Zero. Infact it is not a practical situation.

Page # 73 Tarun Mankad (M.Tech. - IIT Kanpur)


Master Your Physics - NEET / AIIMS KINEMATICS -1

DISPLACEMENT
D

A
O TIME

8. Yes, when the velocity of the object is negative

B
DISPLACEMENT

9.
A
O TIME

10. vA / vB = tan 30° / tan 45° = (1 / 3 ) / 1 = 1/ 3


11. () 12. () 13. (i) infinity (ii) zero 14. ()
15. () 16. () 17. 8 m, 16 m 18. 100m, 60m
19. (i) 170 m, (ii) 125 m, –5 m/s2 20. A, B, D) 21. (B)
22. (D) 23. (B) 24. (D) 25. (A, D)
2
26. (A) 27. 80 m, 2.5 m/s
28. (a) 0.6 m/s2 (b) 50 m (c) 50m
29. (a) 10 m/s (b) 20 m/s, zero, 20 m/s, –20 m/s
30. 100 m, zero 31. x = 5 m, y = 3 m 32. (B) 33. (D)
34. (C) 35. (B) 36. (B)
Practice Problem-6
1. (C) 2. (A) 3. (B) 4. (C) 5. (C)
6. (C) 7. (A)

Practice Problem-7
3. 110 m 4. 0.25 m s–2 5. 136 m 6. 4.5s
7. 2.5 m 8. 2/3 ms–2, 4 ms–2, 65 sec

9. x = 125 m. 10. v  2ad 16. (B) 17. (B)

18. (A) 19. (D) 20. (C) 21. (B)


22. (B) 23. (A) 24. (C) 25. (C)
Practice Problem-8
17. (C) 18. (B) 19. (D) 20. (B) 21. (A)
22. (D) 23. (B) 24. (A) 25. (A) 26. (B)
27. (A) 28. (B) 29. (D) 30. (C) 31. (A)
32. (A)

Page # 74 Tarun Mankad (M.Tech. - IIT Kanpur)


Master Your Physics - NEET / AIIMS KINEMATICS -1
Practice Problem-9

4 2
1. 6 m/s2 2. x == t3 + 3t2+ 2t 3. (i) v = 16 m/s, (ii) + = 125, (iii)  m / s
3

4. v = 3t2 + 6t + 2, a = 6t + 6 5. vt = 36 m/s, a = 14 m/s2 6. v = 18 m/s

2a 2
7. a  8. vt = -9 m/s
3
9. (C) 10.(B) 11. (D) 12. (A) 13. (C)
14. (D) 15. (B) 16.(C) 17.(B)
18.(A) 19.(B)
EXERCISE-2
1. (B) 2. (D) 3. (D) 4. (A) 5. (B) 6. (D) 7. (A) 8. (B)
9. (A) 10. (A) 11. (C) 12. (A) 13. (C) 14. (A) 15. (B) 16. (D)
17. (B) 18. (C) 19. (C) 20. (B) 21. (D) 22. (C) 23. (C) 24. (C)
25. (A) 26. (C) 27. (D) 28. (D) 29. (A) 30. (B) 31. (A) 32. (B)
33. (C) 34. (B) 35. (D) 36. (D) 37. (C) 38. (B) 39. (B) 40. (B)
41. (B) 42. (C) 43. (C) 44. (A) 45. (C) 46. (D) 47. (B) 48. (A)
49. (C) 50. (C) 51. (B) 52. (D) 53. (D) 54. (A) 55. (D) 56. (B)
57. (D) 58. (A) 59. (D) 60. (C) 61. (B) 62. (D) 63. (D) 64. (D)
65. (D) 66. (B) 67. (B) 68. (D) 69. (B) 70. (A) 71. (A) 72. (D)
73. (C) 74. (D) 75. (D) 76. (B) 77. (A) 78. (A) 79. (C) 80. (C)
81. (A) 82. (A) 83. (D) 84. (D) 85. (C) 86. (B) 87. (C) 88. (B)
89. (B) 90. (C) 91. (A) 92. (A) 93. (B) 94. (D) 95. (D) 96. (C)
97. (A) 98. (D) 99. (B) 100. (C) 101. (C) 102. (C) 103. (C) 104. (B)
105. (C) 106. (A) 107. (A) 108. (C) 109. (D) 110. (D) 111. (C) 112. (B)
113. (B) 114. (D) 115. (A) 116. (D) 117.(A, B, C)118. (A) 119. (C) 120. (A)
121. (D) 122. (C) 123. (D) 124. (D) 125. (D) 126. (D) 127. (B) 128. (D)
129. (B) 130. (B) 131. (B) 132. (A) 133. (A) 134. (B) 135. (B) 136. (D)
EXERCISE-3

EXERCISE-4

Page # 75 Tarun Mankad (M.Tech. - IIT Kanpur)

Potrebbero piacerti anche